Easy Office
LCI Learning

CA final #pdf
1347 times
362 KB
Rating:4

Download Other files in Students category

File Content -

Accounting Theory CA Final (Existing) 1 1 ACCOUNTING THEORY Topics Covered:  Accounting Standards in India (Q. No. 1 to 37)  Guidance Notes (Q. No. 38 to 43) Accounting Theory CA Final (Existing) 2 Question 1 Write short notes on the Advantages and disadvantages of setting of Accounting Standards. (4 marks) (May, 2002) (Nov, 2004) Answer The Accounting Standards seek to describe the accounting principles, the valuation techniques and the methods of applying the accounting principles in the preparation and presentation of financial statements so that they may give a true and fair view. The ostensible purpose of the standard setting bodies is to promote the dissemination of timely and useful financial information to investors and certain other parties having an interest in companies’ economic performance. The setting of accounting standards has the following advantages: (i) Standards reduce to a reasonable extent or eliminate altogether confusing variations in the accounting treatments used to prepare financial statements. (ii) There are certain areas where important information are not statutorily required to be disclosed. Standards may call for disclosure beyond that required by law. (iii) The application of accounting standards would, to a limited extent, facilitate comparison of financial statements of companies situated in different parts of the world and also of different companies situated in the same country. However, it should be noted in this respect that differences in the institutions, traditions and legal systems from one country to another give rise to differences in accounting standards practised in different countries. However, there are some disadvantages of setting of accounting standards: (i) Alternative solutions to certain accounting problems may each have arguments to recommend them. Therefore, the choice between different alternative accounting treatments may become difficult. (ii) There may be a trend towards rigidity and away from flexibility in applying the accounting standards. (iii) Accounting standards cannot override the statute. The standards are required to be framed within the ambit of prevailing statutes. Question 2 (a) Briefly indicate the items, which are included in the expression “borrowing cost” as explained in AS 16.(6 marks) (May, 2001) (b) Explain the difference between direct and indirect methods of reporting cash flows from operating activities with reference to Accounting Standard 3( AS 3) revised. (8 marks)(Nov, 2001) (c) Write short note on Effect of Uncertainties on Revenue Recognition. (10 marks) (May, 1997) Answer (a) Borrowing costs: Borrowing costs are interest and other costs incurred by an enterprise in connection with the borrowing of funds. As per para 4 of AS 16 on Borrowing Costs, borrowing costs may include: (a) interest and commitment charges on bank borrowings and other short-term and long-term borrowings; (b) amortisation of discounts or premiums relating to borrowings ; (c) amortisation of ancillary costs incurred in connection with the arrangement of borrowings; (d) finance charges in respect of assets acquired under finance leases or under other similar arrangements; and (e) exchange differences arising from foreign currency borrowings to the extent that they are regarded as an adjustment to interest costs. (b) As per para 18 of AS 3 (Revised) on Cash Flow Statements, an enterprise should report cash flows from operating activities using either: (a) the direct method whereby major classes of gross cash receipts and gross cash payments are disclosed; or (b) the indirect method, whereby net profit or loss is adjusted for the effects of transactions of a non-cash nature, any deferrals or accruals of past or future operating cash receipts or payments, and items of income or expense associated with investing or financing cash flows. The direct method provides information which may be useful in estimating future cash flows and which is not Accounting Theory CA Final (Existing) 3 available under the indirect method and is, therefore, considered more appropriate than the indirect method. Under the direct method, information about major classes of gross cash receipts and gross cash payments may be obtained either: (a) from the accounting records of the enterprise; or (b) by adjusting sales, cost of sales (interest and similar income and interest expense and similar charges for a financial enterprise) and other items in the statement of profit and loss for: (i) changes during the period in inventories and operating receivables and payables: (ii) other non-cash items; and (iii) other items for which the cash effects are investing or financing cash flows. Under the indirect method, the net cash flow from operating activities is determined by adjusting net profit or loss for the effects of: (a) changes during the period in inventories and operating receivables and payables; (b) non-cash items such as depreciation, provisions, deferred taxes, and unrealized foreign exchange gains and losses; and (c) all other items for which the cash effects are investing or financing cash flows. Alternatively, the net cash flow from operating activities may be presented under the indirect method by showing the operating revenues and expenses, excluding non-cash items disclosed in the statement of profit and loss and the changes during the period in inventories and operating receivables and payables. (c) Effect of Uncertainties on Revenue Recognition Para 9 of AS 9 on "Revenue Recognition" deals with the effect of uncertainties on Revenue Recognition. The para states: 1. Recognition of revenue requires that revenue is measurable and at the time of sale or the rendering of the service it would not be unreasonable to expect ultimate collection. 2. Where the ability to assess the ultimate collection with reasonable certainty is lacking at the time of raising any claim, e.g., for escalation of price, export incentives, interest etc. revenue recognition is postponed to the extent of uncertainty involved. In such cases, it may be appropriate to recognise, revenue only when it is reasonably certain that the ultimate collection will be made. When there is uncertainty as to ultimate collection, revenue is recognised at the, time of sale or rendering of service even , though payments are made by instalments. 3. When the uncertainty relating to collectability arises subsequent to the time of sale or rendering of the service, it is more appropriate to make a separate provision to reflect the uncertainty rather than to adjust the amount of revenue originally recorded. 4. An essential criterion for the recognition of revenue is that the consideration receivable for the sale of goods, the rendering of services or from the use by others of enterprise resources is reasonably determinable. When such consideration is not determinable within reasonable limits; the recognition of revenue is postponed. 5. When recognition of revenue is postponed due to the effect of uncertainties, it is considered as revenue of the period in which it is properly recognised. Question 3 How would you deal with the following in the annual accounts of a company for the year ended 31st March, 1996 ? (a) The company has to pay delayed cotton clearing charges over and above the negotiated price for taking delayed delivery of cotton from the Suppliers' Godown. Upto 1994-95, the company has regularly included such charges in the valuation of closing stock. This being in the nature of interest the company has decided to exclude it from closing stock valuation for the year 1995-96. This would result into decrease in profit by Rs. 7.60 lakhs . (3 marks) (b) The company has obtained Institutional Term Loan of Rs. 580 lakhs for modernisation and renovation of its Plant & Machinery. Plant & Machinery acquired under the modernisation scheme and installation completed on 31st March, 1996 amounted to Rs. 406 lakhs, Rs. 58 lakhs has been advanced to suppliers for additional assets and the balance loan of Rs. 116 lakhs has been utilised for working capital purpose. The Accountant is on a dilemma as to how to account for the total interest of Rs. 52.20 lakhs incurred during 1995-96 on the entire Institutional Term Loan of Rs. 580 lakhs. (3 marks) Accounting Theory CA Final (Existing) 4 (c) Fuel surcharge is billed by the State Electricity Board at provisional rates. Final bill for fuel surcharge of Rs. 5.30 lakhs for the period October, 1990 to September, 1994 has been received and paid in February, 1995. (3 marks) (d) The Board of Directors decided on 31.3.1996 to increase the sale price of certain items retrospectively from 1st January, 1996. In view of this price revision with effect from 1st January, 1996, the company has to receive Rs. 15 lakhs from its customers in respect of sales made from 1st January, 1996 to 31st March, 1996 and the Accountant cannot make up his mind whether to include Rs. 15 lakhs in the sales for 1995-96. (3 marks) (May, 1996) Answer (a) Para 29 of AS 5 (Revised) ‘Net Profit or Loss for the Period, Prior Period Items and Changes in Accounting Policies” states that a change in an accounting policy should be made only if the adoption of a different accounting policy is required by statute or for compliance with an accounting standard or if it is considered that the change would result in a more appropriate presentation of the financial statements of an enterprise. Therefore the change in the method of stock valuation is justified in view of the fact that the change is in line with the recommendations of AS 2 (Revised) ‘Valuation of Inventories’ and would result in more appropriate preparation of the financial statements. As per AS 2, this accounting policy adopted for valuation of inventories including the cost formulae used should be disclosed in the finan- cial statements. Also, appropriate disclosure of the change and the amount by which any item in the financial statements is affected by such change is necessary as per AS 1, AS 2 and AS 5. Therefore, the under mentioned note should be given in the annual accounts. "In compliance with the Accounting Standards issued by the ICAl, delayed cotton clearing charges which are in the nature of interest have been excluded from the valuation of closing stock unlike preceding years. Had the company continued the accounting practice followed earlier, the value of closing stock as well as profit before tax for the year would have been higher by Rs. 7.60 lakhs." (b) As per para 6 of AS 16 ‘Borrowing Costs’, borrowing costs that are directly attributable to the acquisition, construction or production of a qualifying asset should be capitalized as part of the cost of that asset. Other borrowing costs should be recognized as an expense in the period in which they are incurred. Borrowing costs should be expensed except where they are directly attributable to acquisition, construction or production of qualifying asset. A qualifying asset is an asset that necessary takes a substantial period of time* to get ready for its intended use or sale. The treatment for total interest amount of Rs. 52.20 lakhs can be given as: Purpose Nature Interest to be charged to profit and loss account Interest to be charged to profit and loss account Rs. in lakhs Rs. in lakhs Modernisation and renovation of plant and machinery Qualifying asset Advance to supplies for additional assets Qualifying asset Working Capital Not a qualifying asset _____ _____ 41.76 10.44 *Accounting Standards Interpretation (ASI) 1 deals with the meaning of expression ‘substantial period of time’. A substantial period of time primarily depends on the facts and circumstances of each case. However, ordinarily, a period of twelve months is considered as substantial period of time unless a shorter or longer period can be justified on the basis of the facts and circumstances of the case. ** It is assumed in the above solution that the modernization and renovation of plant and machinery will take substantial period of time (i.e. more than twelve months). Regarding purchase of additional assets, the nature of additional assets 36.54 580406 20 . 52 * *  5.22 58058 20 . 52 * *  10.44 580 116 20 . 52  Accounting Theory CA Final (Existing) 5 has also been considered as qualifying assts. Alternatively, the plant and machinery and additional assets may be assumed to be non-qualifying assts on the basis that the renovation and installation of additional assets will not take substantial period of time. In that case, the entire amount of interest, Rs. 52.20 lakhs will be recognized as expense in the profit and loss account for year ended 31st March, 1996. (c) The final bill having been paid in February, 1995 should have been accounted for in the annual accounts of the company for the year ended 31st March, 1995. However it seems that as a result of error or omission in the preparation of the financial statements of prior period i.e., for the year ended 31st March 1995, this material charge has arisen in the current period i.e., year ended 31st March, 1996. Therefore it should be treated as 'Prior period item' as per para 16 of AS 5. As per para 19 of AS 5 (Revised), prior period items are normally included in the determination of net profit or loss for the current period. An alternative approach is to show such items in the statement of profit and loss after determination of current net profit or loss. In either case, the objective is to indicate the effect of such items on the current profit or loss. It may be mentioned that it is an expense arising from the ordinary course of business. Although abnormal in amount or infrequent in occurrence, such an expense does not qualify an extraordinary item as per Para 10 of AS 5 (Revised). For better understanding, the fact that power bill is accounted for at provisional rates billed by the state electricity board and final adjustment thereof is made as and when final bill is received may be mentioned as an accounting policy. ' (d) Price revision was effected during the current accounting period 1995-1996. As a result, the company stands to receive Rs. 15 lakhs from its customers in respect of sales made from 1st January, 1996 to 31st March, 1996. If the company is able to assess the ultimate collection with reasonable certainty, then additional revenue arising out of the said price revision may be recognised in 1995-96 vide Para 10 of AS 9. Question 4 Sagar Limited belongs to the engineering industry. The Chief Accountant has prepared the draft accounts for the year ended 31.03.96. You are required to advise the company on the following items from the viewpoint of finalisation of accounts, taking note of the mandatory accounting standards. (a) An audit stock verification during the year revealed that the opening stock of the year was understated by Rs. 3 lakhs due to wrong counting. (b) The company purchased on 01.04.95 special purpose machinery for Rs. 25 lakhs. It received a Central Government Grant for 20% of the price. The machine has an effective life of 10 years. (c) The company undertook a contract for building a crane for Rs. 10 lakhs. As on 31.03.96 it incurred a cost of Rs. 1.5 lakhs and expects that there will be Rs. 9 lakhs more for completing the crane. It has received so far Rs. 1 lakh as progress payment. (d) The company received an actuarial valuation for the first time for its pension scheme which revealed a surplus of Rs. 6 lakhs. It wants to spread the same over the next 2 years by reducing the annual contribution to Rs. 2 lakhs instead of Rs. 5 lakhs. The average remaining life of the employees is estimated to be 6 years. (4  3 =12 Marks)(Nov, 1996) Answer (a) The wrong counting of opening stock of the current year/closing stock of the previous year must have also resulted in lowering of profits of previous year, brought forward to the current year. The adjustments are required to be made in the current year in respect of these errors in the preparation of the financial statements of the prior period and should therefore be treated as prior period adjustments as per AS 5 (Revised). Accordingly, the rectifications relating to both opening stock of the current year and profit brought forward from the previous year should be separately disclosed in the current statement of profit and loss together with their nature and amount in a manner that their impact on current profit or loss can be perceived. (b) AS 12 ‘Accounting for Government Grants’ regards two methods of presentation, of grants related to specific fixed assets, in financial statements as acceptable alternatives. Under the first method, the grant can be shown as a deduction from the gross book value of the machinery in arriving at its book value. The grant is thus recognised in the profit and loss statement over the useful life of a depreciable asset by way of a reduced depreciation charge. Under the second method, it can be treated as deferred income which should be recognised in the profit and loss statement over the useful life of 10 years in the proportions in which depreciation on machinery will be charged. The deferred income pending its apportionment to profit and loss account should be disclosed in the balance sheet with a suitable description e.g., ‘Deferred government grants' to be shown after 'Reserves and Surplus' but before 'Secured Loans'. The following should also be disclosed: (i) The accounting policy adopted for government grants, including the methods of presentation in the financial statements; Accounting Theory CA Final (Existing) 6 (ii) The nature and extent of government grants recognised in the financial statement. (c) Para 21 of AS 7 (Revised) ‘Construction Contracts’ provides that when the outcome of a construction contract can be estimated reliably, contract revenue and contract costs associated with the construction contract should be recognized as revenue and expenses respectively with reference to the stage of completion of the contract activity at the reporting date. As per para 32 of the standard, during the early stages of a contact it is often the case that the outcome of the contract cannot be estimated reliably. Nevertheless, it may be probable that the enterprise will recover the contract costs incurred. Therefore, contract revenue is recognized only to the extent of costs incurred that are expected to be recovered. As the outcome of the contract cannot be estimated reliably, no profit is recognised. Para 35 of the standard states that when it is probable that the total contacts costs will exceed total contract revenue, the expected loss should be recognised as an expense immediately. Thus the foreseeable loss of Rs. 50,000 (expected cost Rs. 10.5 lakhs less contract revenue Rs. 10 lakhs) should be recognized as an expense in the year ended 31st March, 1996. Also, the following disclosures should be given in the financial statements: (a) the amount of contract revenue recognized as revenue in the period; (b) the aggregate amount of costs incurred and loss recognized upto the reporting date; (c) amount of advances received; (d) amount of retentions; and (e) gross amount due from/due to customers Amount  (d) As per AS 15 ‘Accounting for Retirement Benefits in the Financial Statements of Employers’, the surplus amount of Rs. 6 lakhs can be either credited to the profit and loss account of the current year or, alternatively, spread over a period not more than the expected remaining life of the participating employees i.e. 6 years. This change relating to actuarial valuation for its pension scheme should be treated as a change in an accounting policy and disclosed in accordance with AS 5 (Revised). The financial statements should disclose: (a) the method for determination of these retirement benefit costs; (b) whether the actuarial valuation was made at the end of the period or at an earlier date (also specify date); and (iii) the method by which the accrual for the period has been determined (if the same is not based on the report of the actuary). Note: AS 15 was revised in March, 2005. According to para 92 of AS 15 (Revised 2005) ‘Employee Benefits’, actuarial gains and losses should be recognized immediately in the statement of profit and loss as income or expense. Therefore, surplus amount of Rs. 6 lakhs is required to be credited to the profit and loss statement of the current year. Question 5 A firm of contractors obtained a contract for construction of bridges across river Revathi. The following details are available in the records kept for the year ended 31st March, 1997. (Rs. in lakhs) Total Contract Price 1,000 Work Certified 500 Work not Certified 105 Estimated further Cost to Completion 495 Progress Payment Received 400 To be Received 140 The firm seeks your advice and assistance in the presentation of accounts keeping in view the requirements of AS 7 (Revised) issued by your institute. (15 marks) (Nov, 1997) Answer (a) Amount of foreseeable loss (Rs in lakhs) Total cost of construction (500 + 105 + 495) 1,100 Less: Total contract price 1,000                                                              Amount due from/to customers = contract costs + Recognised profits – Recognised losses – Progress billings = 1.5 + Nil – 0.5 – 1.0 = Nil. Accounting Theory CA Final (Existing) 7 Total foreseeable loss to be recognized as expense 100 According to para 35 of AS 7 (Revised 2002), when it is probable that total contract costs will exceed total contract revenue, the expected loss should be recognized as an expense immediately. (b) Contract work-in-progress i.e. cost incurred to date are Rs. 605 lakhs (Rs in lakhs) Work certified 500 Work not certified 105 605 This is 55% (605/1,100  100) of total costs of construction. (c) Proportion of total contract value recognised as revenue as per para 21 of AS 7 (Revised). 55% of Rs. 1,000 lakhs = Rs. 550 lakhs (d) Amount due from/to customers = Contract costs + Recognised profits – Recognised losses – (Progress payments received + Progress payments to be received) = [605 + Nil – 100 – (400 + 140)] Rs. in lakhs = [605 – 100 – 540] Rs. in lakhs Amount due to customers = Rs. 35 lakhs The amount of Rs. 35 lakhs will be shown in the balance sheet as liability. (e) The relevant disclosures under AS 7 (Revised) are given below: Rs. in lakhs Contract revenue 550 Contract expenses 605 Recognised profits less recognized losses (100) Progress billings (400 + 140) 540 Retentions (billed but not received from contractee) 140 Gross amount due to customers 35 Question 6 In preparing the financial statements of R Ltd. for the year ended 31st March, 1998, you come across the following information. State with reasons, how you would deal with them in the financial statements: (a) An unquoted long term investment is carried in the books at a cost of Rs. 2 lakhs. The published accounts of the unlisted company received in May, 1998 showed that the company was incurring cash losses with declining market share and the long term investment may not fetch more than Rs. 20,000. (b) The company invested 100 lakhs in April, 1998 in the acquisition of another company doing similar business, the negotiations for which had started during the financial year. (c) There was a major theft of stores valued at Rs. 10 lakhs in the preceding year which was detected only during current financial year (97-98). (15 marks)(May, 1998) Answer As it is stated in the question that financial statements for the year ended 31st March, 1998 are under preparation, the views have been given on the basis that the financial statements are yet to be completed and approved by the Board of Directors. (a) Investments classified as long term investments should be carried in the financial statements at cost. However, provision for diminution shall be made to recognise a decline, other than temporary, in the value of the investments, such reduction being determined and made for each investment individually. Para 17 of AS 13 ‘Accounting for Investments’ states that indicators of the value of an investment are obtained by reference to its market value, the investee's assets and results and the expected cash flows from the investment. On these bases, the facts of the given case clearly suggest that the provision for diminution should be made to reduce the carrying amount of long term investment to Rs. 20,000 in the financial statements for the year ended 31st March, 1998. (b) Para 3.2 of AS 4 (Revised) defines "Events occurring after the balance sheet date" as those significant events, both favourable Accounting Theory CA Final (Existing) 8 and unfavourable, that occur between the balance sheet date and the date on which the financial statements are approved by the Board of Directors in the case of a company. Accordingly, the acquisition of another company is an event occurring after the balance sheet date. However no adjustment to assets and liabilities is required as the event does not affect the determination and the condition of the amounts stated in the financial statements for the year ended 31st March, 1998. Applying para 15 which clearly states that/disclosure should be made in the report of the approving authority of those events occurring after the balance sheet date that represent material changes and commitments affecting the financial position of the enterprise, the investment of Rs. 100 lakhs in April, 1998 in the acquisition of another company should be disclosed in the report of the Board of Directors to enable users of financial statements to make proper evaluations and decisions. (c) Due to major theft of stores in the preceding year (1996-97) which was detected only during the current financial year (1997- 98), there was overstatement of closing stock of stores in the preceding year. This must have also resulted in the overstatement of profits of previous year, brought forward to the current year. The adjustments are required to be made in the current year as 'Prior Period Items' as per AS 5 (Revised) on Net Profit or Loss for the Period, Prior Period Items and Changes in Accounting Policies. Accordingly, the adjustments relating to both opening stock of the current year and profit brought forward from the previous year should be separately disclosed in the statement of profit and loss together with their nature and amount in a manner that their impact on the current profit or loss can be perceived. Note: Alternatively, it may be assumed that in the preceding year, the value of stock of stores as found out by physical verification of stocks was considered in the preparation of financial statements of the preceding year. In such a case, only the disclosure as to the theft and the resulting loss is required in the notes to the accounts for the current year i.e, year ended 31st March, 1998. Question 7 (a) A Limited Company closed its accounting year on 30.6.98 and the accounts for that period were considered and approved by the board of directors on 20th August, 1998. The company was engaged in laying pipe line for an oil company deep beneath the earth. While doing the boring work on 1.9.1998 it had met a rocky surface for which it was estimated that there would be an extra cost to the tune of Rs. 80 lakhs. You are required to state with reasons, how the event would be dealt with in the financial statements for the year ended 30.6.98. (b) X Co. Ltd., has obtained an Institutional Loan of Rs. 680 lakhs for modernisation and renovation of its plant & machiney, Plant & machinery acquired under the modernisation scheme and installation completed on 31.3.98 amounted to Rs. 520 lakhs, 30 lakhs has been advanced to suppliers for additional assets and the balance loan of Rs. 130 lakhs has been utilized for working capital purpose. The total interest paid for the above loan amounted to Rs. 62 lakhs during 1997-98. You are required to state how the interest on the institutional loan is to be accounted for in the year 1997-98. (c) Y Co. Ltd., used certain resources of X Co. Ltd. In return X Co. Ltd. received Rs. 10 lakhs and Rs. 15 lakhs as interest and royalties respective from Y Co. Ltd. during the year 1997-98. You are required to state whether and on what basis these revenues can be recognised by X Co. Ltd. (d) A Ltd. purchased fixed assets costing Rs. 3,000 lakhs on 1.1.98 and the same was fully financed by foreign currency loan (U.S. Dollars) payable in three annual equal instalments. Exchange rates were 1 Dollar = Rs. 40.00 and Rs. 42.50 as on 1.1.98 and 31.12.98 respectively. First instalment was paid on 31.12.98. The entire difference in foreign exchange has been capitalized. You are required to state, how these transactions would be accounted for. (e) A Limited Company finds that the stock sheets as on 31.3.97 had included twice an item the cost of which was Rs. 20,000. You are asked to suggest, how the error would be dealt with in the accounts of the year ended 31.3.98 (3+ 4+3+3+3 = 16 marks)(May, 1999) Answer (a) Para 3.2 of AS 4 (Revised) on Contingencies and Events Occurring after the Balance Sheet Date defines 'events occurring after the balance sheet date' as 'significant events, both favourable and unfavourable, that occur between the balance sheet date and the date on which financial statements are approved by the Board of Directors in the case of a company'. The given case is discussed in the light of the above mentioned definition and requirements given in paras 13-15 of the said AS 4 (Revised). In this case the incidence, which was expected to push up cost became evident after the date of approval of the accounts. So that was not an 'event occurring after the balance sheet date'. However, this may be mentioned in the Directors’ Report. Accounting Theory CA Final (Existing) 9 (b) The treatment for total interest amount of Rs. 68 lakhs can be given as follows: Purpose Nature Interest to be capitalized Interest to be charged to profit and loss account Rs. in lakhs Rs. in lakhs Modernisation and renovation of plant and machinery Qualifying asset Advance to suppliers for additional assets Qualifying asset  Working Capital Not a qualifying asset _____ = 11.85 50.15 11.85 For details of para 6 of AS 16 ‘Borrowing Costs’, Qualifying asset, substantial period of time, refer Answer 3(b). (c) As per para 13 of AS 9 on Revenue Recognition, revenue arising from the use by others of enterprise resources yielding interest and royalties should only be recognised when no significant uncertainty as to measurability or collectability exists. These revenues are recognised on the following bases: (i) Interest: on a time proportion basis taking into account the amount outstanding and the rate applicable. (ii) Royalties: on an accrual basis in accordance with the terms of the relevant agreement. (d) As per para 13 of AS 11 (Revised 2003) ‘The Effects of Changes in Foreign Exchange Rates’, exchange differences arising on the settlement of monetary items or on reporting an enterprise’s monetary items at rates different from those at which they were initially recorded during the period, or reported in previous financial statements, should be recognized as income or expenses in the period in which they arise. Thus exchange differences arising on repayment of liabilities incurred for the purpose of acquiring fixed assets are recognized as income or expense. Calculation of Exchange Difference: Dollars US lakhs 75 40 Rs.lakhs 3,000 Rs. loan currency Foreign  Exchange difference = 75 lakhs US Dollars  (42.50 – 40.00) = Rs. 187.50 lakhs (including exchange loss on payment of first instalment) Therefore, entire loss due to exchange differences amounting Rs. 187.50 lakhs should be charged to profit and loss account for the year. (e) The error in the recording of closing stock of the year ended 31st March, 1997 must have also resulted in overstatement of profits of previous year, brought forward to the current year ended 31st March, 1998. Vide para 4 of AS 5 (Revised) on Net Profit or Loss for the Period, Prior Period Items and Changes in Accounting Policies, the rectifications as required in the current year are 'Prior Period Items'. Accordingly, Rs. 20,000 should be deducted from opening stock in the profit and loss account. And Rs. 20,000 should be charged as prior period adjustment in the profit and loss account for the year ended 31st March 1998 in accordance with para 15 of AS 5 (Revised) which requires that the nature and amount of prior period items should be separately disclosed in the statement of profit and loss in a manner that their impact on the current profit or loss can be perceived. Question 8 (i) Advise P Co. Ltd. about the treatment of the following in the Final Statement of Accounts for the year ended 31st March, 2000.                                                                Alternatively, the plant and machinery and additional assets may be assumed to be non-qualifying assets. In that case, the entire amount of interest Rs. 62 lakhs will be recognized as expense in the profit and loss account for the year ended 31st March, 1998. 47.41 680520 62   2.74 68030 62   680130 62  Accounting Theory CA Final (Existing) 10 A claim lodged with the Railways in March, 1997 for loss of goods of Rs. 2,00,000 had been passed for payment in March, 2000 for Rs. 1,50,000. No entry was passed in the books of the Company, when the claim was lodged. (3 marks) (May 1996, May, 2000) (ii) The notes to accounts of X Ltd. for the year 1999-2000 include the following: “Interest on bridge loan from banks and Financial Institutions and on Debentures specifically obtained for the Company’s Fertiliser Project amounting to Rs. 1,80,80,000 has been capitalized during the year, which includes approximately Rs. 1,70,33,465 capitalised in respect of the utilization of loan and debenture money for the said purpose.” Is the treatment correct? Briefly comment. (6 marks)(May, 2000) Answer (i) Prudence suggests non-consideration of claim as an asset in anticipation. So receipt of claims is generally recognised on cash basis. Para 9.2 of AS 9 on Revenue Recognition states that where the ability to assess the ultimate collection with reasonable certainty is lacking at the time of raising any claim, revenue recognition is postponed to the extent of uncertainty involved. Para 9.5 of AS 9 states that when recognition of revenue is postponed due to the effect of uncertainties, it is considered as revenue of the period in which it is properly recognised. In this case it may be assumed that collectability of claim was not certain in the earlier periods. This is supposed from the fact that only Rs. 1,50,000 were collected against a claim of Rs. 2,00,000. So this transaction can not be taken as a Prior Period Item. In the light of revised AS 5, it will not be treated as extraordinary item. However, para 12 of AS 5 (Revised) states that when items of income and expense within profit or loss from ordinary activities are of such size, nature, or incidence that their disclosure is relevant to explain the performance of the enterprise for the period, the nature and amount of such items should be disclosed separately. Accordingly, the nature and amount of this item should be disclosed separately as per para 12 of AS 5 (Revised). (ii) The treatment done by the company is not in accordance with AS 16 ‘Borrowing Costs’. As per para 10 of AS 16, to the extent that funds are borrowed specifically for the purpose of obtaining a qualifying asset, the amount of borrowing costs eligible for capitalisation on that asset should be determined as the actual borrowing costs incurred on that borrowing during the period. Hence, the capitalisation of borrowing costs should be restricted to the actual amount of interest expenditure i.e. Rs. 1,70,33,465. Thus, there is an excess capitalisation of Rs. 10,46,535. This has resulted in overstatement of profits by Rs. 10,46,535 and amount of fixed assets has also gone up by this amount. Question 9 (i) T. Ltd. imported fixed assets worth Rs. 1,000 lacs on 1.4.1999, when the exchange rate was Rs. 40 per US $. The assets were fully financed by foreign currency loan repayable in five equal annual installments. As on 31.3.2000, the first installment was paid at the Exchange Rate of Rs. 42. (ii) The company’s fixed assets stood at Rs. 3,000 lacs as on 1.4.1999. It provides depreciation at 10% per annum under the WDV method. However it noticed that about Rs. 500 lacs worth of non-imported assets acquired on 1.4.1999 will be obsolete in 2 years time. It wants to write off these assets over 2 years. (iii) A few days after the beginning of the year, the company acquired assets for Rs. 500 lacs on which it received a government grant of 10%. Prepare a schedule as on 31.3.2000 in respect of the above three categories of assets and support the schedule with relevant accounting standards. (8 marks) (Nov, 2000) Answer In the books of T Ltd. Schedule of Fixed Assets as on 31st March, 2000 (Amount in Rs. lacs) Fixed Assets Gross Block (at cost) Depreciation Net Block As at 1.4.1999 Additions Deductions As at 31.3.2000 Up to 31.3.1999 For the year On Deductions Total upto 31.3.2000 As at 31.3.2000 As at 31.3.1999 Imported Assets – 1,000 – 1,000 – 100 – 100 900 – Accounting Theory CA Final (Existing) 11 Non - Imported Assets (acquired on 1.4.1999) – 500 – 500 – 250 – 250 250 – Other Assets 3,000 450 – 3,450 – 345 – 345 3,105 3,000 Total 3,000 1,950 – 4,950 – 695 – 695 4,255 3,000 (1) As per para 13 of AS 11 (Revised 2003) ‘The Effects of Changes in Foreign Exchange Rates’, exchange differences arising on repayment of liabilities incurred for the purpose of acquiring fixed assets are recognized as income/expense in the period in which they arise. Calculation of Exchange Difference: Foreign currency loan = 40 Rs. lacs 1,000 Rs. = 25 lacs US $ Exchange difference = 25 lacs US $ × (42 – 40) = Rs. 50 lacs (including exchange loss on payment of first installment) Thus, exchange loss of Rs. 50 lakhs should be recognized as expense in the profit and loss account for the year ended 31st March, 2000. (2) It was noticed that about Rs. 500 lacs worth of non-imported assets acquired on 1.4.1999 will be obsolete in two years time. Hence, these assets have been written off at the rate of 50%. (3) Para 14 of AS 12 on Accounting for Government Grants regards two methods of presentation of grants related to specific fixed assets in financial statements. Under the first method which has been applied in the given case, the grant is shown as a deduction from the gross value of the fixed assets in arriving at its book value. Thus, only 90% of the cost of fixed assets has been shown as addition after adjusting the grant amount. Alternatively, the grant can be treated as a deferred income which should be recognised in the profit and loss statement over the useful life of fixed assets in the proportions in which depreciation on the assets will be charged. Note: As regards fixed assets standing at Rs. 3,000 lacs as on 1.4.1999, in the absence of information in respect of cost and depreciation amount provided upto 31.3.1999, the entire given amount has been shown under gross block as at 1.4.1999. Question 10 State with reference to accounting standard, how will you value the inventories in the following cases: (i) Raw material was purchased at Rs. 100 per kilo. Price of raw material is on the decline. The finished goods in which the raw material is incorporated is expected to be sold at below cost. 10,000 kgs. of raw material is on stock at the year end. Replacement cost is Rs. 80 per kg. (ii) In a production process, normal waste is 5% of input. 5,000 MT of input were put in process resulting in a wastage of 300 MT. Cost per MT of input is Rs. 1,000. The entire quantity of waste is on stock at the year end. (iii) Per kg. of finished goods consisted of: Material cost Rs. 100 per kg. Direct labour cost Rs. 20 per kg. Direct variable production overhead Rs. 10 per kg. Fixed production charges for the year on normal capacity of one lakh kgs. is Rs. 10 lakhs. 2,000 kgs. of finished goods are on stock at the year end. (3 x 4 = 12 marks)(Nov, 2000) Answer (a) (i) As per para 24 of AS 2 (Revised) on Valuation of Inventories, materials and other supplies held for use in the production of inventories are not written down below cost if the finished product in which they will be incorporated are expected to be sold at or above cost. However, when there has been a decline in the price of materials and it is estimated that the cost of the finished products will exceed net realisable value, the materials are written down Accounting Theory CA Final (Existing) 12 to net realisable value. In such circumstances, the replacement cost of the materials may be the best available measure of their net realisable value. Hence, in the given case, the stock of 10,000 kgs of raw material will be valued at Rs. 80 per kg. The finished goods, if on stock, should be valued at cost or net realisable value whichever is lower. (ii) As per para 13 of AS 2 (Revised), abnormal amounts of waste materials, labour or other production costs are excluded from cost of inventories and such costs are recognised as expenses in the period in which they are incurred. In this case, normal waste is 250 MT and abnormal waste is 50 MT. The cost of 250 MT will be included in determining the cost of inventories (finished goods) at the year end. The cost of abnormal waste amounting to Rs. 50,000 (50 MT x Rs. 1,000) will be charged in the profit and loss statement. (iii) In accordance with paras 8 and 9 of AS 2 (Revised), the costs of conversion include a systematic allocation of fixed and variable production overheads that are incurred in converting materials into finished goods. The allocation of fixed production overheads for the purpose of their inclusion in the costs of conversion is based on the normal capacity of the production facilities. Thus, cost per kg. of finished goods can be computed as follows: Rs. Material cost 100 Direct labour cost 20 Direct variable production overhead 10 Fixed production overhead       1,00,00010,00,000 . Rs 10 ___ 140 Thus, the value of 2,000 kgs of finished goods on stock at the yearend will be Rs. 2,80,000 (2,000 kgs. x Rs. 140). Question 11 From the following Summary Cash Account of X Ltd. prepare Cash Flow Statement for the year ended 31st March, 2001 in accordance with AS 3 (Revised) using the direct method. The company does not have any cash equivalents. Summary Cash Account for the year ended 31.3.2001 Rs. ’000 Rs. ’000 Balance on 1.4.2000 50 Payment to Suppliers 2,000 Issue of Equity Shares 300 Purchase of Fixed Assets 200 Receipts from Customers 2,800 Overhead expense 200 Sale of Fixed Assets 100 Wages and Salaries 100 Taxation 250 Dividend 50 Repayment of Bank Loan 300 _____ Balance on 31.3.2001 150 3,250 3,250 (8 marks)(Nov, 2001) Answer X Ltd. Cash Flow Statement for the year ended 31st March, 2001 (Using the direct method) Rs. ’000 Rs. ’000 Accounting Theory CA Final (Existing) 13 Cash flows from operating activities Cash receipts from customers 2,800 Cash payment to suppliers (2,000) Cash paid to employees (100) Cash payments for overheads (200) Cash generated from operations 500 Income tax paid (250) Net cash from operating activities 250 Cash flows from investing activities Payment for purchase of fixed assets (200) Proceeds from sale of fixed assets 100 Net cash used in investing activities (100) Cash flows from financing activities Proceeds from issuance of equity shares 300 Bank loan repaid (300) Dividend paid (50) Net cash used in financing activities (50) Net increase in cash 100 Cash at beginning of the period 50 Cash at end of the period 150 Question 12 Answer the following questions by quoting the relevant Accounting Standard: (i) During the year 2001-2002, a medium size manufacturing company wrote down its inventories to net realisable value by Rs. 5,00,000. Is a separate disclosure necessary? (ii) A Limited company has been including interest in the valuation of closing stock. In 2001-2002, the management of the company decided to follow AS 2 and accordingly interest has been excluded from the valuation of closing stock. This has resulted in a decrease in profits by Rs. 3,00,000. Is a disclosure necessary? If so, draft the same. (iii) A company signed an agreement with the Employees Union on 1.9.2001 for revision of wages with retrospective effect from 30.9.2000. This would cost the company an additional liability of Rs. 5,00,000 per annum. Is a disclosure necessary for the amount paid in 2001-02 ? (12 marks) (May, 2002) Answer (i) Although the case under consideration does not relate to extraordinary item, but the nature and amount of such item may be relevant to users of financial statements in understanding the financial position and performance of an enterprise and in making projections about financial position and performance. Para 12 of AS 5 (Revised in 1997) on Net Profit or Loss for the Period, Prior Period Items and Changes in Accounting Policies states that : “When items of income and expense within profit or loss from ordinary activities are of such size, nature or incidence that their disclosure is relevant to explain the performance of the enterprise for the period, the nature and amount of such items should be disclosed separately.” Circumstances which may give to separate disclosure of items of income and expense in accordance with para 12 of AS 5 include the write-down of inventories to net realisable value as well as the reversal of such write-downs. (ii) As per AS 5 (Revised), change in accounting policy can be made for many reasons, one of these is for compliance with an accounting standard. In the instant case, the company has changed its accounting policy in order to conform with the AS 2 (Revised) on Valuation of Inventories. Therefore, a disclosure is necessary in the following lines by way of notes to the annual accounts for the year 2001-2002. “To be in conformity with the Accounting Standard on Valuation of Inventories issued by ICAI, interest has been excluded from the valuation of closing stock unlike preceding years. Had the same principle been followed in previous years, profit for the year and its corresponding effect on the year end net assets would have been higher by Rs. 3,00,000.” (iii) It is given that revision of wages took place on 1st September, 2001 with retrospective effect from 30.9.2000. Therefore wages payable for the half year from 1.10.2000 to 31.3.2001 cannot be taken as an error or omission in the preparation of Accounting Theory CA Final (Existing) 14 financial statements and hence this expenditure cannot be taken as a prior period item. Additional wages liability of Rs. 7,50,000 (for 1½ years @ Rs. 5,00,000 per annum) should be included in current year’s wages. It may be mentioned that additional wages is an expense arising from the ordinary activities of the company. Although abnormal in amount, such an expense does not qualify as an extraordinary item. However, as per Para 12 of AS 5 (Revised), when items of income and expense within profit or loss from ordinary activities are of such size, nature or incidence that their disclosure is relevant to explain the performance of the enterprise for the period, the nature and amount of such items should be disclosed separately. Question 13 A company obtained term loan during the year ended 31st March, 2002 in an extent of Rs. 650 lakhs for modernisation and development of its factory. Buildings worth Rs. 120 lakhs were completed and Plant and Machinery worth Rs. 350 lakhs were installed by 31st March, 2002. A sum of Rs. 70 lakhs has been advanced for Assets the installation of which is expected in the following year. Rs. 110 lakhs has been utilised for Working Capital requirements. Interest paid on the loan of Rs. 650 lakhs during the year 2001 – 2002 amounted to Rs. 58.50 lakhs. How should the interest amount be treated in the Accounts of the Company? (6 marks) (Nov, 2002) Answer The treatment for total interest amount of Rs. 58.50 lakhs can be given as follows: Purpose Nature Interest to be capitalized Interest to be charged to profit and loss account Rs. in lakhs Rs. in lakhs Buildings Qualifying asset Plant and machinery Qualifying asset Advance to suppliers for additional assets Qualifying asset Working capital Not a qualifying asset ____ ____ 48.6 9.9 For details of para 6 of AS 16 ‘Borrowing Costs’, Qualifying asset, Substantial Period of Time, refer Question 3(b). Question 14 In the context of relevant Accounting Standards, give your comments on any four of the following matters for the financial year ending on 31.3.2002. (a) Assets and liabilities and income and expenditure items in respect of foreign branches are translated into Indian rupees at the prevailing rate of exchange at the end of the year. The resultant exchange differences in the case of profit, is carried to other Liabilities Account and the Loss, if any, is charged to revenue. (b) Leave encashment benefit is accounted for as per “Pay-as-you-go” method. (c) Increase in pension liability on account of wage revision in 1999 – 2000 is being provided for in 5 instalments commencing from that year. The remaining liability of Rs. 300 lakhs as re-determined in actuarial valuation will be provided for in the next 2 years. (d) A Pharma Company spent Rs. 33 lakhs during the accounting year ended 31st March, 2002 on a research project to develop a drug to treat “AIDS”. Experts are of the view that it may take four years to establish whether the drug will be effective or not and even if found effective it may take two to three more years to produce the medicine, which can be marketed. The company wants to treat the expenditure as deferred revenue expenditure. 10.8 650120 5 . 58   31.5 650350 5 . 58   6.3 65070 5 . 58   9.9 650110 5 . 58   Accounting Theory CA Final (Existing) 15 (e) While preparing its final accounts for the year ended 31st March, 2002 Rainbow Limited created a provision for Bad and Doubtful debts are 2% on trade debtors. A few weeks later the company found that payments from some of the major debtors were not forthcoming. Consequently the company decided to increase the provision by 10% on the debtors as on 31st March, 2002 as the accounts were still open awaiting approval of the Board of Directors. Is this to be considered as an extra-ordinary item or prior period item ? (4  4 = 16 marks) (Nov, 2002) Answer (a) The financial statements of an integral foreign operation (for example, dependent foreign branches) should be translated using the principles and procedures described in paragraphs 8 to 16 of AS 11 (Revised 2003). The individual items in the financial statements of a foreign operation are translated as if all its transactions had been entered into by the reporting enterprise itself. Individual items in the financial statements of the foreign operation are translated at the actual rate on the date of transaction. For practical reasons, a rate that approximates the actual rate at the date of transaction is often used, for example, an average rate for a week or a month may be used for all transactions in each foreign currency during the period. The foreign currency monetary items (for example cash, receivables, payables) should be reported using the closing rate at each balance sheet date. Non-monetary items (for example, fixed assets, inventories, investments in equity shares) which are carried in terms of historical cost denominated in a foreign currency should be reported using the exchange date at the date of transaction. Thus the cost and depreciation of the tangible fixed assets is translated using the exchange rate at the date of purchase of the asset if asset is carried at cost. If the fixed asset is carried at fair value, translation should be done using the rate existed on the date of the valuation. The cost of inventories is translated at the exchange rates that existed when the cost of inventory was incurred and realizable value is translated applying exchange rate when realizable value is determined which is generally closing rate. Exchange difference arising on the translation of the financial statements of integral foreign operation should be charged to profit and loss account. Exchange difference arising on the translation of the financial statement of foreign operation may have tax effect which should be dealt as per AS 22 ‘Accounting for Taxes on Income’. Thus, the treatment by the management of translating all assets and liabilities; income and expenditure items in respect of foreign branches at the prevailing rate at the year end and also the treatment of resultant exchange difference is not in consonance with AS 11 (Revised 2003). Note: For the purpose of translation of assets, liabilities, income and expenditure items of foreign operations, AS 11 (Revised 2003) classifies the foreign operation into two types – Integral foreign operation, Non-integral foreign operation. Integral foreign operation is a foreign operation, the activities of which are an integral part of those of the reporting enterprise. Non-integral foreign operation is a foreign operation that is not an integral foreign operation. The above answer has been given on the basis that the foreign branches referred in the question are integral foreign operations. (b) As per para 12 of AS 15 on 'Accounting for Retirement Benefits in the Financial Statements of Employers', the cost of retirement benefits to an employer results from receiving services from the employees who are entitled to receive such benefits. Consequently, the cost of retirement benefits is accounted for in the period during which these services are rendered. Accounting for retirement benefit cost only when employees retire or receive benefits payments (i.e. as per pay as you go method) does not achieve the objective of allocation of those costs to the periods in which the services were rendered. Hence, the treatment of leave encashment benefit by the management is not in consonance with AS 15. Note: AS 15 was revised in March, 2005. AS 15 (revised 2005) covers the leave encashment benefits under the category of short-term employee benefits. Accumulating short-term compensated absences (i.e. earned leaves) are those that are carried forward and can be used for future periods if the current period’s entitlement is not used in full [para 13 of AS 15(Revised)]. Earned leaves which are encashable on retirement or resignation are vesting (which entitle employees to receive cash payment for unused entitlements on leaving the enterprise) accumulating compensated absences. ‘An enterprise should measure the expected cost of accumulating compensated absences as the additional amount that the enterprise expects to pay as a result of the unused entitlement that has accumulated at the balance sheet date’. [Para 14 of AS 15 (Revised)]. (c) Revision of wages and consequential increase in pension liability of employer is not a prior period item as it has not arisen out of errors or omissions of previous year. It is also not an extraordinary item as defined in AS 5 on Net profit or Loss for the Period, Prior Period Items and Changes in Accounting Policies. It is an expense arising out of the ordinary activity of the enterprise. Therefore, it should have been charged during the year 1999-2000, and disclosed separately. The treatment of deferring to two years, Rs. 30 crores remaining pension liability as redetermined by actuarial valuation Accounting Theory CA Final (Existing) 16 is also not in consonance with AS 15 relating to Accounting for Retirement Benefits in the Financial Statements of Employers. As per para 29 of AS 15, any alternations in the retirement benefit costs arising from changes in the actuarial method used or assumptions adopted should be charged or credited to the statement of profit and loss as they arise in accordance with AS 5, “Prior Period and Extraordinary Items and Changes in Accounting Policies”. Additionally, a change in the actuarial method used should be treated as a change in an accounting policy and disclosed in accordance with AS 5. Note: AS 15 was revised in March, 2005. As per para 92 of AS 15 (Revised 2005) ‘Employee Benefits’, actuarial gains and losses should be recognized immediately in the statement of profit and loss as income or expense. (d) As per para 41 of AS 26 ‘Intangible Assets’, no intangible asset arising from research (or from the research phase of an internal project) should be recognized. Expenditure on research (or on the research phase of an internal project) should be recognized as an expense when it is incurred. Thus the company cannot treat the expenditure as deferred revenue expenditure. The entire amount of Rs. 33 lakhs spent on research project should be charged as an expense in the year ended 31st March, 2002. (e) The preparation of financial statements involve making estimates which are based on the circumstances existing at the time when the financial statements are prepared. It may be necessary to revised an estimate in a subsequent period if there is a change in the circumstances on which the estimate was based. Revision of an estimate does not bring the resulting amount within the definition either of prior period item or of an extraordinary item [para 21, AS 5 (Revised)]. In the given case, Rainbow Limited created a provision for bad and doubtful debts at 2% on trade debtors while preparing its final accounts for the year ended 31st March, 2002. Subsequently, the company decided to increase the provision by 10%. As per AS 5 (Revised), this change in estimate is neither a prior period item nor an extraordinary item. However, as per para 27 of AS 5 (Revised), a change in accounting estimate which has a material effect in the current period should be disclosed and quantified. Any change in an accounting estimate which is expected to have a material effect in later periods should also be disclosed. Question 15 From the Books of Bharati Ltd., following informations are available as on 1.4.2001 and 1.4.2002: (1) Equity Shares of Rs. 10 each 1,00,000 (2) Partly paid Equity Shares of Rs. 10 each Rs. 5 paid 1,00,000 (3) Options outstanding at an exercise price of Rs. 60 for one equity share Rs. 10 each. Average Fair Value of equity share during both years Rs. 75 10,000 (4) 10% convertible preference shares of Rs. 100 each. Conversion ratio 2 equity shares for each preference share 80,000 (5) 12% convertible debentures of Rs. 100. Conversion ratio 4 equity shares for each debenture 10,000 (6) 10% dividend tax is payable for the years ending 31.3.2003 and 31.3.2002. (7) On 1.10.2002 the partly paid shares were fully paid up (8) On 1.1.2003 the company issued 1 bonus share for 8 shares held on that date. Net profit attributable to the equity shareholders for the years ending 31.3.2003 and 31.3.2002 were Rs. 10,00,000. Calculate : (i) Earnings per share for years ending 31.3.2003 and 31.3.2002. (ii) Diluted earnings per share for years ending 31.3.2003 and 31.3.2002. (iii) Adjusted earnings per share and diluted EPS for the year ending 31.3.2002, assuming the same information for previous year, also assume that partly paid shares are eligible for proportionate dividend only. (14 marks) (May, 2003) Answer (i) Earnings per share Year ended Year ended Accounting Theory CA Final (Existing) 17 31.3.2003 31.3.2002 Net profit attributable to equity shareholders Rs. 10,00,000 Rs. 10,00,000 Weighted average number of equity shares 2,00,000 1,50,000 [(W.N. 1) – without considering bonus issue for the year ended 31.3.2002] Earning per share Rs. 5 Rs. 6.667 (ii) Diluted earnings per share Options are most dilutive as their earnings per incremental share is nil. Hence, for the purpose of computation of diluted earnings per share, options will be considered first. 12% convertible debentures being second most dilutive will be considered next and thereafter convertible preference shares will be considered (as per W.N. 2). Year ended 31.3.2003 Year ended 31.3.2002 Net profit attributable to equity shareholders Rs. No. of equity shares Net Profit attributable per share Rs. No. of equity shares (without considering bonus issue) Net Profit attributable per share Rs. As reported (for years ended 31.3.2003 and 31.3.2002) 10,00,000 2,00,000 5 1,50,000 6.667 Options ________ 2,000 2,000 10,00,000 2,02,000 4.95 Dilutive 1,52,000 6.579 Dilutive 12% Convertible Debentures 84,000 40,000 40,000 10,84,000 2,42,000 4.48 Dilutive 1,92,000 5.646 Dilutive 10% Convertible Preference Shares 8,80,000 1,60,000 1,60,000 19,64,000 4,02,000 4.886 Anti- Dilutive 3,52,000 5.58 Dilutive Since diluted earnings per share is increased when taking the convertible preference shares into account (Rs. 4.48 to Rs. 4.886), the convertible preference shares are anti-dilutive and are ignored in the calculation of diluted earnings per share for the year ended 31.3.2003. Therefore, diluted earnings per share for the year ended 31st March, 2003 is Rs. 4.48. For the year ended 31st March, 2002, Options, 12% Convertible debentures and Convertible preference shares will be considered dilutive and diluted earnings per share will be taken as Rs. 5.58. Year ended 31.3.2003 Year ended 31.3.2003 Diluted earnings per Share 4.48 5.58 (iii) Adjusted earnings per share and diluted earnings per share for the year ending 31.3.2002. Net profit attributable to equity shareholders Rs. 10,00,000 Weighted average number of equity shares [(W.N. 1) – considering bonus issue] 1,75,000 Adjusted earnings per share Rs. 5.714 Calculation of adjusted diluted earnings per share Accounting Theory CA Final (Existing) 18 Net profit attributable to equity shareholders Rs. No. of equity shares (after considering bonus issue) Net profit attributable per share Rs. As reported 10,00,000 1,75,000 5.714 Options ________ 2,000 10,00,000 1,77,000 5.65 Dilutive 12% Convertible Debentures 84,000 40,000 10,84,000 2,17,000 4.995 Dilutive 10% Convertible Preference Shares 8,80,000 1,60,000 19,64,000 3,77,000 5.21 Anti –Dilutive Since diluted earnings per share is increased when taking the convertible preference shares into account (from Rs. 4.995 to Rs. 5.21), the convertible preference shares are anti-dilutive and are ignored in the calculation of diluted earnings per share. Therefore, adjusted diluted earnings per share for year ended 31.3.2002 is Rs. 4.995. Adjusted diluted earnings per share Rs. 4.995 Working Notes: 1. Weighted average number of equity shares 31.3.2003 No. of Shares 31.3.2002 No. of Shares (a) Fully paid equity shares 1,00,000 1,00,000 (b) Partly paid equity shares* 50,000 Partly paid equity shares 25,000 Fully paid equity shares 50,000 (Partly paid shares converted into fully paid up on 1.10.2002) (c) Bonus Shares** 25,000 _______ Weighted average number of equity shares 2,00,000 1,50,000 (without considering bonus issue for year ended 31.3.2002) Bonus Shares Weighted average number of equity shares (after considering bonus issue for year ended 31.3.2002) 25,000 1,75,000 *Since partly paid equity shares are entitled to participate in dividend to the extent of amount paid, 1,00,000 equity shares of Rs. 10 each, Rs. 5 paid up will be considered as 50,000 equity shares for the year ended 31st March, 2002. On 1st October, 2002 the partly paid shares were converted into fully paid up. Thus, the weighted average equity shares (for six months ended 30th September, 2002) will be calculated as 50,000 × 126= 25,000 shares Weighted average shares (for six months ended 31st March, 2003) will be calculated as 1,00,000 × 126 = 50,000 shares ** Total number of fully paid shares on 1st January, 2003 Fully paid shares on 1st April, 2002 1,00,000 Partly paid shares being made fully paid up on 1st October, 2002 1,00,000 2,00,000 The company issued 1 bonus share for 8 shares held on 1st January, 2003. Thus 2,00,000/8 = 25,000 bonus shares will be issued. Accounting Theory CA Final (Existing) 19 Bonus is an issue without consideration, thus it will be treated as if it had occured prior to the beginning of 1st April, 2001, the earliest period reported. 2. Increase in earnings attributable to equity shareholders on conversion of potential equity shares Increase in earnings (1) Increase in number of equity shares (2) Earnings per incremental share (3) = (1) ÷ (2) Rs. Rs. Options Increase in earnings Nil No. of incremental shares issued for no consideration [10,000 × (75 – 60)/75] 2,000 Nil Convertible Preference Shares Increase in net profit attributable to equity shareholders as adjusted by attributable dividend tax [(Rs. 10 × 80,000) + 10% (Rs. 10 × 80,000)] 8,80,000 No. of incremental shares (2 × 80,000) 1,60,000 5.50 12% Convertible Debentures Increase in net profit [(Rs.10,00,000 × 0.12 × (1 – 0.30)]* 84,000 No. of incremental shares (10,000 × 4) 40,000 2.10 * Tax rate has been taken at 30% in the absence of any information in the question. Question 16 A Ltd. acquired 25% of shares in B Ltd. as on 31.3.2002 for Rs. 3 lakhs. The Balance Sheet of B Ltd. as on 31.3.2002 is given below: Rs. Share Capital 5,00,000 Reserves and Surplus 5,00,000 10,00,000 Fixed Assets 5,00,000 Investments 2,00,000 Current Assets 3,00,000 10,00,000 During the year ended 31.3.2003 the following are the additional information available: (i) A Ltd. received dividend from B Ltd., for the year ended 31.3.2002 at 40% from the Reserves. (ii) B Ltd., made a profit after tax of Rs. 7 lakhs for the year ended 31.3.2003. (iii) B Ltd., declared a dividend @ 50% for the year ended 31.3.2003 on 30.4.2003. A Ltd. is preparing Consolidated Financial Statements in accordance with AS – 21 for its various subsidiaries. Calculate: (i) Goodwill if any on acquisition of B Ltd.’s shares. Accounting Theory CA Final (Existing) 20 (ii) How A Ltd., will reflect the value of investment in B Ltd., in the Consolidated Financial Statements? (iii) How the dividend received from B Ltd. will be shown in the Consolidated Financial Statements? (6 marks)(May, 2003) Answer In terms of AS 23 B Ltd. will be considered as an associate company of A Ltd. as shares acquired represent to more than 20%. (i) Calculation of Goodwill Rs.in lakhs Cost of investment 3.00 Less: Share in the value of Equity of B.Ltd. as at the date of investment [25% of Rs.10 lakhs (Rs.5 lakhs + Rs. 5 lakhs)] 2.50 Goodwill 0.50 (ii) A Ltd. Consolidated Profit and Loss Account for the year ended 31st March, 2003 Rs. in lakhs By Share of profits in B Ltd. 1.75 By Dividend received from B Ltd. 0.50 Transfer to investment A/c 0.50 Nil (iii) A Ltd. Consolidated Balance Sheet as on 31.3.2003 Rs. in lakhs Investment in B Ltd. Share in B Ltd.'s Equity 2.50 Less: Dividend received 0.50 2.00 Share of Profit for year 2002 – 2003 1.75 3.75 Add: Goodwill 0.50 4.25 Working Notes: 1. Dividend received from B Ltd. amounting to Rs. 0.50 lakhs will be reduced from investment value in the books of A Ltd. However goodwill will not change. 2. B Ltd. made a profit of Rs. 7 lakhs for the year ended 31st March, 2003. A Ltd.’s share in the profits of Rs. 7 lakhs is Rs. 1.75 lakhs. Investment in B Ltd. will be increased by Rs. 1.75 lakhs and consolidated profit and loss account of A Ltd. will be credited with Rs. 1.75 lakhs in the consolidated financial statement of A Ltd. 3. Dividend declared on 30th April, 2003 will not be recognised in the consolidated financial statements of A Ltd. Question 17 XYZ Ltd., has undertaken a project for expansion of capacity as per the following details: Plan Actual Rs. Rs. April, 2002 2,00,000 2,00,000 May, 2002 2,00,000 3,00,000 Accounting Theory CA Final (Existing) 21 June, 2002 10,00,000 – July, 2002 1,00,000 – August, 2002 2,00,000 1,00,000 September, 2002 5,00,000 7,00,000 The company pays to its bankers at the rate of 12% p.a., interest being debited on a monthly basis. During the half year company had Rs. 10 lakhs overdraft upto 31st July, surplus cash in August and again overdraft of over Rs. 10 lakhs from 1.9.2002. The company had a strike during June and hence could not continue the work during June. Work was again commenced on 1st July and all the works were completed on 30th September. Assume that expenditure were incurred on 1st day of each month. Calculate: (i) Interest to be capitalised. (ii) Give reasons wherever necessary. Assume: (a) Overdraft will be less, if there is no capital expenditure. (b) The Board of Directors based on facts and circumstances of the case has decided that any capital expenditure taking more than 3 months as substantial period of time. (8 marks) (May, 2003) Answer (a) XYZ Ltd. Month Actual Expenditure Interest Capitalised Cumulative Amount Rs. Rs. Rs. April, 2002 2,00,000 2,000 2,02,000 May, 2002 3,00,000 5,020 5,07,020 June, 2002 – 5,070 5,12,090 Note 2 July, 2002 – 5,120 5,17,210 August, 2002 1,00,000 – 6,17,210 Note 3 September, 2002 7,00,000 10,000 13,27,210 Note 4 13,00,000 27,210 13,27,210 Note: 1. There would not have been overdraft, if there is no capital expenditure. Hence, it is a case of specific borrowing as per AS 16 on Borrowing Costs. 2. The company had a strike in June and hence could not continue the work during June. As per para 14 (c) of AS 16, the activities that are necessary to prepare the asset for its intended use or sale are in progress. The strike is not during extended period. Thus during strike period, interest need to be capitalised. 3. During August, the company did not incur any interest as there was surplus cash in August. Therefore, no amount should be capitalised during August as per para 14(b) of AS 16. 4. During September, it has been taken that actual overdraft is Rs. 10 lakhs only. Hence, only Rs. 10,000 interest has been capitalised even though actual expenditure exceeds Rs. 10 lakhs. Alternatively, interest may be charged on total amount of (Rs. 6,17,210 + Rs. 7,00,000 = 13,17,210) for the month of September, 2002 as it is given in the question that overdraft was over Rs. 10 lakhs from 1.9.2002 and not exactly Rs. 10 lakhs. In that case, interest amount Rs. 13,172 will be capitalised for the month of September. Question 18 Briefly explain, as per relevant Accounting Standard: Accounting Theory CA Final (Existing) 22 (a) TVSM company has taken a Transit Insurance Policy. Suddenly in the year 2002-2003 the percentage of accident has gone up to 7% and the company wants to recognise insurance claim as revenue in 2002-2003 in accordance with relevant Accounting Standards. Do you agree? (b) SCL Ltd., sells agriculture products to dealers. One of the condition of sale is that interest is payable at the rate of 2% p.m., for delayed payments. Percentage of interest recovery is only 10% on such overdue outstanding due to various reasons. During the year 2002-2003 the company wants to recognise the entire interest receivable. Do you agree? (c) ABC Ltd. was making provision for non-moving stocks based on no issues for the last 12 months upto 31.3.2002. The company wants to provide during the year ending 31.3.2003 based on technical evaluation: Total value of stock Rs. 100 lakhs Provision required based on 12 months issue Rs. 3.5 lakhs Provision required based on technical evaluation Rs. 2.5 lakhs Does this amount to change in Accounting Policy? Can the company change the method of provision? (d) XYZ is an export oriented unit and was enjoying tax holiday upto 31.3.2002. No provision for deferred tax liability was made in accounts for the year ended 31.3.2002. While finalising the accounts for the year ended 31.3.2003, the Accountant says that the entire deferred tax liability upto 31.3.2002 and current year deferred tax liability should be routed through Profit and Loss Account as the relevant Accounting Standard has already become mandatory from 1.4.2001. Do you agree? (16 marks)(May, 2003) Answer (a) AS 9 on Revenue Recognition defines revenue as ‘gross inflow of cash, receivables or other consideration arising in the course of the ordinary activities of the enterprise from the sale of goods, from the rendering of services and from the use by others of enterprise resources yielding interest, royalties and dividends’. To recognise revenue AS 9 requires that revenue arises from ordinary activities and that it is measurable and there should be no uncertainty. As per para 9.2 of the Standard, where the ability to assess the ultimate collection with reasonable certainty is lacking at the time of raising any claim, revenue recognition is postponed to the extent of uncertainty involved. In such cases, it may be appropriate to recognise revenue only when it is reasonably certain that the ultimate collection will be made. In the given case, TVSM company wants to suddenly recognise Insurance claim because it has increased over the previous year. But, there are uncertainties involved in the settlement of the claim. Also, the claim does not seem to be in the course of ordinary activity of the company. Hence, TVSM company is not advised to recognise the Insurance claim as revenue. (b) As per para 9.2 of AS 9 on Revenue Recognition, where the ability to assess the ultimate collection with reasonable certainty is lacking at the time of raising any claim, e.g. for escalation of price, export incentives, interest etc, revenue recognition is postponed to the extent of uncertainty involved. In such cases, it may be appropriate to recognise revenue only when it is reasonably certain that the ultimate collection will be made. Where there is no uncertainty as to ultimate collection, revenue is recognised at the time of sale or rendering of service even though payments are made by instalments. Thus, SCL Ltd. cannot recognise the interest amount unless the company actually receives it. 10% rate of recovery on overdue outstandings is also an estimate and is not certain. Hence, the company is advised to recognise interest receivable only on receipt basis. (c) The decision of making provision for non-moving stocks on the basis of technical evaluation does not amount to change in accounting policy. Accounting policy of a company may require that provision for non-moving stocks should be made. The method of estimating the amount of provision may be changed in case a more prudent estimate can be made. In the given case, considering the total value of stock, the change in the amount of required provision of non-moving stock from Rs.3.5 lakhs to Rs.2.5 lakhs is also not material. The disclosure can be made for such change in the following lines by way of notes to the accounts in the annual accounts of ABC Ltd. for the year 2002-03: “The company has provided for non-moving stocks on the basis of technical evaluation unlike preceding years. Had the same method been followed as in the previous year, the profit for the year and the corresponding effect on the year end net assets would have been higher by Rs.1 lakh.” Accounting Theory CA Final (Existing) 23 (d) Paragraph 33 of AS 22 on “Accounting For Taxes on Income” relates to the transitional provisions. It says, “On the first occasion that the taxes on income are accounted for in accordance with this statement, the enterprise should recognise, in the financial statements, the deferred tax balance that has accumulated prior to the adoption of this statement as deferred tax asset/liability with a corresponding credit/charge to the revenue reserves, subject to the consideration of prudence in case of deferred tax assets. Further Paragraph 34 lays down, “For the purpose of determining accumulated deferred tax in the period in which this statement is applied for the first time, the opening balances of assets and liabilities for accounting purposes and for tax purposes are compared and the differences, if any, are determined. The tax effects of these differences, if any, should be recognised as deferred tax assets or liabilities, if these differences are timing differences.” Therefore, in the case of XYZ, even though AS 22 has come into effect from 1.4.2001, the transitional provisions permit adjustment of deferred tax liability/asset upto the previous year to be adjusted from opening reserve. In other words, the deferred taxes not provided for alone can be adjusted against opening reserves. Provision for deferred tax asset/liability for the current year should be routed through profit and loss account like normal provision. Question 19 PQR Ltd.'s accounting year ends on 31st March. The company made a loss of Rs. 2,00,000 for the year ending 31.3.2001. For the years ending 31.3.2002 and 31.3.2003, it made profits of Rs. 1,00,000 and Rs. 1,20,000 respectively. It is assumed that the loss of a year can be carried forward for eight years and tax rate is 40%. By the end of 31.3.2001, the company feels that there will be sufficient taxable income in the future years against which carry forward loss can be set off. There is no difference between taxable income and accounting income except that the carry forward loss is allowed in the years ending 2002 and 2003 for tax purposes. Prepare a statement of Profit and Loss for the years ending 2001, 2002 and 2003. (4 marks) (Nov, 2003) Answer Statement of Profit and Loss 31.3.2001 31.3.2002 31.3.2003 Rs. Rs. Rs. Profit (Loss) (2,00,000) 1,00,000 1,20,000 Less: Current tax (8,000) Deferred tax: Tax effect of timing differences originating during the year 80,000 Tax effect of timing differences reversed/adjusted during the year (40,000) (40,000) Profit (loss) after tax effect (1,20,000) 60,000 72,000 Question 20 (a) J Ltd. purchased machinery from K Ltd. on 30.09.2001. The price was Rs. 370.44 lakhs after charging 8% Sales-tax and giving a trade discount of 2% on the quoted price. Transport charges were 0.25% on the quoted price and installation charges come to 1% on the quoted price. A loan of Rs. 300 lakhs was taken from the bank on which interest at 15% per annum was to be paid. Expenditure incurred on the trial run was Materials Rs. 35,000, Wages Rs. 25,000 and Overheads Rs. 15,000. Machinery was ready for use on 1.12.2001. However, it was actually put to use only on 1.5.2002. Find out the cost of the machine and suggest the accounting treatment for the expenses incurred in the interval between the dates 1.12.2001 to 1.5.2002. The entire loan amount remained unpaid on 1.5.2002. Accounting Theory CA Final (Existing) 24 (b) State, how you will deal with the following matters in the accounts of U Ltd. for the year ended 31st March, 2003 with reference to Accounting Standards: (i) The company finds that the stock sheets of 31.3.2002 did not include two pages containing details of inventory worth Rs. 14.5 lakhs. (ii) The company had spent Rs. 45 lakhs for publicity and research expenses on one of its new consumer product, which was marketed in the accounting year 2002-2003, but proved to be a failure. (7 + 8 = 15 marks)(Nov, 2003) Answer (a) Rs. (in Lakhs) (Rs. in Lakhs) Quoted price (refer to working note) 350.00 Less: 2% Trade Discount 7.00 343.00 Add: 8% Sales tax (8% × Rs. 343 lakhs) 27.44 370.44 Transport charges (0.25% × Rs. 350 lakhs) 0.88 (approx.) Installation charges (1% × Rs. 350 lakhs) 3.50 Financing cost (15% on Rs.300 Lakhs) for the period 30.9.2001 to 1.12.2001 7.50 Trial Run Expenses Material 0.35 Wages 0.25 Overheads 0.15 0.75 Total cost 383.07 Interest on loan for the period 1.12.2001 to 1.05.2002 is Rs. 300 lakhs 125 10015   = Rs.18.75 lakhs This expenditure may be charged to Profit and Loss Account or deferred for amortization between say three to five years. Assumed that no other expenses are incurred on the machine during this period. Working Note: Let the quoted price ‘X’ Less: Trade Discount 0.02X. Actual Price = 0.98X. Sale Tax @8% = 1.08 × 0.98X lakhs 350 Rs. 0.98 1.08lakhs 370.44 Rs. X or    (b) (i) Paragraph 4 of Accounting Standard 5 on Net Profit or Loss for the Period, Prior Period Items and Changes in Accounting Policies, defines Prior Period items as "income or expenses which arise in the current period as a result of errors or omissions in the preparation of the financial statements of one or more prior periods”. Rectification of error in stock valuation is a prior period item vide Para 4 of AS 5. Rs.14.5 lakhs must be added to the opening stock of 1/4/2002. It is also necessary to show Rs. 14.5 lakhs as a prior period adjustment in the Profit and loss Account below the line. Separate disclosure of this item as a prior period item is required as per Para 15 of AS 5. (ii) In the given case, the company spent Rs. 45 lakhs for publicity and research of a new product which was marketed but proved to be a failure. It is clear that in future there will be no related further revenue/benefit because of the failure of the product. Thus according to paras 41 to 43 of AS 26 ‘Intangible Assets’, the company should charge the total amount of Rs. 45 lakhs as an expense in the profit and loss account. Question 21 Accounting Theory CA Final (Existing) 25 (a) On 1st December, 2002, Vishwakarma Construction Co. Ltd. undertook a contract to construct a building for Rs. 85 lakhs. On 31st March, 2003 the company found that it had already spent Rs. 64,99,000 on the construction. Prudent estimate of additional cost for completion was Rs. 32,01,000. What amount should be charged to revenue in the final accounts for the year ended 31st March, 2003 as per provisions of Accounting Standard 7 (Revised)? (b) While preparing its final accounts for the year ended 31st March, 2003 a company made a provision for bad debts @ 5% of its total debtors. In the last week of February, 2003 a debtor for Rs. 2 lakhs had suffered heavy loss due to an earthquake; the loss was not covered by any insurance policy. In April, 2003 the debtor became a bankrupt. Can the company provide for the full loss arising out of insolvency of the debtor in the final accounts for the year ended 31st March, 2003? (5+ 5 = 10 marks)(Nov, 2003) Answer (a) Rs. Cost incurred till 31st March, 2003 64,99,000 Prudent estimate of additional cost for completion 32,01,000 Total cost of construction 97,00,000 Less: Contract price 85,00,000 Total foreseeable loss 12,00,000   According to para 35 of AS 7 (Revised 2002), the amount of Rs. 12,00,000 is required to be recognized as an expense. Contract work in progress = 97,00,000 100 64,99,000 Rs.= 67% Proportion of total contract value recognized as turnover as per para 21 of AS 7 (Revised) on Construction Contracts. = 67% of Rs.85,00,000 = Rs.56,95,000. (b) As per paras 8.2 and 13 of Accounting Standard 4 on Contingencies and Events Occurring after the Balance Sheet Date, Assets and Liabilities should be adjusted for events occurring after the balance sheet date that provide additional evidence to assist estimation of amounts relating to conditions existing at the balance sheet date. So full provision for bad debt amounting to Rs. 2 lakhs should be made to cover the loss arising due to the insolvency in the Final Accounts for the year ended 31 st March, 2003. It is because earthquake took place before the balance sheet date. Had the earthquake taken place after 31 st March, 2003, then mere disclosure required as per para 15, would have been sufficient. Question 22 (a) At the end of the financial year ending on 31st December, 2003, a company finds that there are twenty law suits outstanding which have not been settled till the date of approval of accounts by the Board of Directors. The possible outcome as estimated by the Board is as follows: Probability Loss (Rs.) In respect of five cases (Win) 100%  Next ten cases (Win) 60%  Lose (Low damages) 30% 1,20,000 Lose (High damages) 10% 2,00,000 Remaining five cases Win 50%  Lose (Low damages) 30% 1,00,000 Lose (High damages) 20% 2,10,000 Outcome of each case is to be taken as a separate entity. Ascertain the amount of contingent loss and the accounting treatment in respect thereof. (b) Z Ltd. presents the following information for the year ending 31.03.2002 and 31.03.2003 from which you are required to Accounting Theory CA Final (Existing) 26 calculate the Deferred Tax Asset/Liability assuming tax rate of 30% and state how the same should be dealt with as per relevant accounting standard. 31.03.2002 31.03.2003 Rs. (lakhs) Rs. (lakhs) Depreciation as per books 4,010.10 4,023.54 Unabsorbed carry forward business loss and depreciation allowance 2,016.60 4,110.00 Disallowance under Section 43B of Income tax Act, 1961 518.35 611.45 Deferred Revenue Expenses 4.88  Provision for Doubtful Debts 282.51 294.35 Z Ltd. had incurred a loss of Rs. 504 lakhs for the year ending 31.03.2003 before providing for Current Tax of Rs. 26.00 lakhs. (4 + 6 = 10 marks)(May, 2004) Answer (a) According to AS 29 ‘Provisions, Contingent Liabilities and Contingent Assets’, contingent liability should be disclosed in the financial statements if following conditions are satisfied: (i) There is a present obligation arising out of past events but not recognized as provision. (ii) It is not probable that an outflow of resources embodying economic benefits will be required to settle the obligation. (iii) The possibility of an outflow of resources embodying economic benefits is also remote. (iv) The amount of the obligation cannot be measured with sufficient reliability to be recognized as provision. In this case, the probability of winning of first five cases is 100% and hence, question of providing for contingent loss does not arise. The probability of winning of next ten cases is 60% and for remaining five cases is 50%. As per AS 29, we make a provision if the loss is probable. As the loss does not appear to be probable and the possibility of an outflow of resources embodying economic benefits is not remote rather there is reasonable possibility of loss, therefore disclosure by way of note should be made. For the purpose of the disclosure of contingent liability by way of note, amount may be calculated as under: Expected loss in next ten cases = 30% of Rs. 1,20,000 + 10% of Rs. 2,00,000 = Rs. 36,000 + Rs. 20,000 = Rs. 56,000 Expected loss in remaining five cases = 30% of Rs. 1,00,000 + 20% of Rs. 2,10,000 = Rs. 30,000 + Rs. 42,000 = Rs. 72,000 To disclose contingent liability on the basis of maximum loss will be highly unrealistic. Therefore, the better approach will be to disclose the overall expected loss of Rs. 9,20,000 (Rs. 56,000  10 + Rs. 72,000  5) as contingent liability. (b) Rs. in lakhs 31.3.2002 31.3.2003 Carried Forward Business Loss and Depreciation Allowance 2,016.60 4,110.00 Add: Disallowance under Section 43 B of Income Tax Act,1961 518.35 611.45 Provision for Doubtful Debts 282.51 294.35 2,817.46 5,015.80 Less: Depreciation 4,010.10 4,023.54 () 1,192.64 992.26 Accounting Theory CA Final (Existing) 27 Less: Deferred Revenue Expenditure 4.88  Timing Differences () 1,197.52 992.26 Deferred Tax Liability 359.26 Deferred Tax Asset 297.68 Where an enterprise has unabsorbed depreciation or carry forward of losses under tax laws, deferred tax assets should be recognized only to the extent that there is virtual certainty supported by convincing evidence that future taxable income will be available against which such deferred tax assets can be realized. The existence of unabsorbed depreciation or carry forward of losses is strong evidence that future taxable income may not be available. Deferred Tax Asset of Rs. 297.68 lakhs should not be recognized as an asset as per para 17 of AS 22 on ‘Accounting for Taxes on Income’. Deferred Tax Liability of Rs. 359.26 lakhs should be disclosed under a separate heading in the balance sheet of Z Ltd., separately from current assets and current liabilities. Question 23 (a) X Co. Ltd. supplied the following information. You are required to compute the basic earning per share: (Accounting year 1.1.2002 – 31.12.2002) Net Profit : Year 2002 : Rs. 20,00,000 : Year 2003 : Rs. 30,00,000 No. of shares outstanding prior to Right Issue : 10,00,000 shares Right Issue : One new share for each four outstanding i.e., 2,50,000 shares. Right Issue price – Rs. 20 Last date of exercise rights – 31.3.2003. Fair rate of one Equity share immediately prior to exercise of rights on 31.3.2003 : Rs. 25 (b) A Ltd. Leased a machinery to B Ltd. on the following terms: (Rs. in Lakhs) Fair value of the machinery 20.00 Lease term 5 years Lease Rental per annum 5.00 Guaranteed Residual value 1.00 Expected Residual value 2.00 Internal Rate of Return 15% Depreciation is provided on straight line method @ 10% per annum. Ascertain unearned financial income and necessary entries may be passed in the books of the Lessee in the First year. (c) The following particulars are stated in the Balance Sheet of M/s Exe Ltd. as on 31.03.2003: (Rs. in Lakhs) Deferred Tax Liability (Cr.) 20.00 Deferred Tax Assets (Dr.) 10.00                                                               It is assumed that the deferred revenue expenditure is actually incurred during the year ended 31st March, 2002 and it is fully allowed under the Income Tax Act. Accounting Theory CA Final (Existing) 28 The following transactions were reported during the year 2003-04: (i) Tax Rate 50% (ii) Depreciation – As per Books 50.00 Depreciation – for Tax purposes 30.00 There were no addition to Fixed Assets during the year. (iii) Items disallowed in 2002-03 and allowed for Tax purposes in 2003-04 10.00 (iv) Interest to Financial Institutions accounted in the Books on accrual basis, but actual payment was made on 30.09.2004 20.00 (v) Donations to Private Trusts made in 2003-04 10.00 (vi) Share issue expenses allowed under 35(D) of the I.T. Act, 1961 for the year 2003-04 (1/10th of Rs. 50.00 lakhs incurred in 1999-2000) 5.00 (vii) Repairs to Plant and Machinery Rs. 100.00 lakhs was spread over the period 2003-04 and 2004-05 equally in the books. However, the entire expenditure was allowed for Income-tax purposes. Indicate clearly the impact of above items in terms of Deferred Tax liability/Deferred Tax Assets and the balances of Deferred Tax Liability/Deferred Tax Asset as on 31.03.2004. (8 + 8 + 4 = 20 marks)(Nov, 2004) Answer (a) Computation of Basic Earnings Per Share (as per paragraphs 10 and 26 of AS 20 on Earnings Per Share) Year 2002 Year 2003 Rs. Rs. EPS for the year 2002 as originally reported = year the during g outstandin shares equity of number average Weightedrs shareholde equity to le attributab year the of profit Net = (Rs. 20,00,000 / 10,00,000 shares) 2.00 EPS for the year 2002 restated for rights issue = [Rs. 20,00,000 / (10,00,000 shares  1.04 )] 1.92 (approx.) EPS for the year 2003 including effects of rights issue 9/12) shares (12,50,000 3/12) 1.04 shares (10,00,00030,00,000 Rs.     shares 11,97,50030,00,000 Rs. 2.51 (approx.) Working Notes: 1. Computation of theoretical ex-rights fair value per share exercise the in issued shares of Number exercise to prior g outstandin shares of Numberexercise from received amount Total rights of exercise to prior y immediatel shares g outstandin all of value Fair   shares 2,50,000 shares 10,00,000shares 2,50,000 20 Rs. shares 10,00,000 25 . Rs                                                                  Refer working note 2. Accounting Theory CA Final (Existing) 29 24 Rs. shares 2,50,000 10 3,00,00,00 Rs.   2. Computation of adjustment factor share per value rights - ex heoretical Trights of exercise to prior share per value Fair  (approx.) 1.04 1) Note Working (Refer 24 . Rs25 Rs.   (b) Computation of Unearned Finance Income As per AS 19 on Leases, unearned finance income is the difference between (a) the gross investment in the lease and (b) the present value of minimum lease payments under a finance lease from the standpoint of the lessor; and any unguaranteed residual value accruing to the lessor, at the interest rate implicit in the lease. where : (a) Gross investment in the lease is the aggregate of (i) minimum lease payments from the stand point of the lessor and (ii) any unguaranteed residual value accruing to the lessor. Gross investment = Minimum lease payments + Unguaranteed residual value = (Total lease rent + Guaranteed residual value) + Unguaranteed residual value = [(Rs. 5,00,000  5 years) + Rs. 1,00,000] + Rs. 1,00,000 = Rs. 27,00,000 (b) Table showing present value of (i) Minimum lease payments (MLP) and (ii) Unguaranteed residual value (URV). Year MLP inclusive of URV Internal rate of return (Discount factor 15%) Present Value Rs. Rs. 1 5,00,000 .8696 4,34,800 2 5,00,000 .7561 3,78,050 3 5,00,000 .6575 3,28,750 4 5,00,000 .5718 2,85,900 5 5,00,000 .4972 2,48,600 1,00,000 .4972 49,720 (guaranteed residual value) ________ 17,25,820 (i) 1,00,000 .4972 49,720 (ii) (unguaranteed residual value) ________ (i) + (ii) 17,75,540 (b) Unearned Finance Income = (a) – (b) = Rs. 27,00,000 – Rs. 17,75,540 = Rs. 9,24,460 Journal Entries in the books of B Ltd. Rs. Rs. At the inception of lease Machinery account Dr. 17,25,820                                                              As per para 11 of AS 19, the lessee should recognise the lease as an asset and a liability at an amount equal to the fair value of the leased asset at the inception of lease. However, if the fair value of the leased asset exceeds the present Accounting Theory CA Final (Existing) 30 To A Ltd.’s account 17,25,820* (Being lease of machinery recorded at present value of MLP) At the end of the first year of lease Finance charges account (Refer Working Note) Dr. 2,58,873 To A Ltd.’s account 2,58,873 (Being the finance charges for first year due) A Ltd.’s account Dr. 5 ,00,000 To Bank account 5,00,000 (Being the lease rent paid to the lessor which includes outstanding liability of Rs. 2,41,127 and finance charge of Rs. 2,58,873) Depreciation account Dr. 1,72,582 To Machinery account 1,72,582 (Being the depreciation provided @ 10% p.a. on straight line method) Profit and loss account Dr. 4,31,455 To Depreciation account 1,72,582 To Finance charges account 2,58,873 (Being the depreciation and finance charges transferred to profit and loss account) Working Note: Table showing apportionment of lease payments by B Ltd. between the finance charges and the reduction of outstanding liability. Year Outstanding liability (opening balance) Lease rent Finance charge Reduction in outstanding liability Outstanding liability (closing balance) Rs. Rs. Rs. Rs. Rs. 1 17,25,820 5,00,000 2,58,873 2,41,127 14,84,693 2 14,84,693 5,00,000 2,22,704 2,77,296 12,07,397 3 12,07,397 5,00,000 1,81,110 3,18,890 8,88,507 4 8,88,507 5,00,000 1,33,276 3,66,724 5,21,783 5 5,21,783 5,00,000 78,267 5,21,783 1,00,050* 8,74,230 17,25,820 * The difference between this figure and guaranteed residual value (Rs. 1,00,000) is due to approximation in computing the interest rate implicit in the lease. (c) Impact of various items in terms of deferred tax liability/deferred tax asset. Transactions Analysis Nature of difference Effect Amount                                                                                                                                                                                                                         value of minimum lease payments from the standpoint of lessee, the amount recorded should be the present value of these minimum lease payments. Therefore, in this case, as the fair value of Rs. 20,00,000 is more than the present value amounting Rs. 17,25,820, the machinery has been recorded at Rs. 17,25,820 in the books of B Ltd. (the lessee) at the inception of the lease. According to para 13 of the standard, at the inception of the lease, the asset and liability for the future lease payments are recognised in the balance sheet at the same amounts. Accounting Theory CA Final (Existing) 31 Difference in depreciation Generally, written down value method of depreciation is adopted under IT Act which leads to higher depreciation in earlier years of useful life of the asset in comparison to later years. Responding timing difference Reversal of DTL Rs. 20 lakhs  50% = Rs. 10 lakhs Disallowances, as per IT Act, of earlier years Tax payable for the earlier year was higher on this account. Responding timing difference Reversal of DTA Rs. 10 lakhs  50% = Rs. 5 lakhs Interest to financial institutions It is allowed as deduction under section 43B of the IT Act, if the payment is made before the due date of filing the return of income (i.e. 31st October, 2004). No timing difference Not applicable Not applicable Donation to private trusts Not an allowable expenditure under IT Act. Permanent difference Not applicable Not applicable Share issue expenses Due to disallowance of full expenditure under IT Act, tax payable in the earlier years was higher. Responding timing difference Reversal of DTA Rs. 5 lakhs  50% = Rs. 2.5 lakhs Repairs to plant and machinery Due to allowance of full expenditure under IT Act, tax payable of the current year will be less. Originating timing difference Increase in DTL Rs. 50 lakhs  50% = Rs. 25 lakhs Deferred Tax Liability Account Dr. Cr. Rs. in lakhs Rs. in lakhs 31.3.2004 To Profit and Loss account (Depreciation) 10.00 1.4.2003 By By Balance b/d Profit and Loss Account 20.00 25.00 To Balance c/d 35.00 (Repairs to plant) ____ 45.00 45.00 1.4.2004 By Balance b/d 35.00 Deferred Tax Asset Account Dr. Cr. Rs. in lakhs Rs. in lakhs 1.4.2003 To Balance b/d 10.00 31.3.2004 By Profit and Loss Account: Accounting Theory CA Final (Existing) 32 Items disallowed in 2002-03 and allowed as per I.T. Act in 2003-04 5.00 Share issue expenses 2.50 ____ By Balance c/d 2.50 10.00 10.00 1.4.2004 To Balance b/d 2.50 Question 24 (a) An equipment is leased for 3 years and its useful life is 5 years. Both the cost and the fair value of the equipment are Rs. 3,00,000. The amount will be paid in 3 instalments and at the termination of lease lessor will get back the equipment. The unguaranteed residual value at the end of 3 years is Rs. 40,000. The (internal rate of return) IRR of the investment is 10%. The present value of annuity factor of Re. 1 due at the end of 3rd year at 10% IRR is 2.4868. The present value of Re. 1 due at the end of 3rd year at 10% rate of interest is 0.7513. (i) State with reason whether the lease constitutes finance lease. (ii) Calculate unearned finance income. (b) Intelligent Corporation (I Corp.) is dealing in seasonal products. The quarterly sales pattern of the product is given below: Quarter I II III IV Ending 31st March 30th June 30th September 31st December - - - -  For the First quarter ending 31st March, 2005, ICorp. gives you the following information: Rs. crores Sales 50 Salary and other expenses 30 Advertisement expenses (routine) 02 Administrative and selling expenses 08 While preparing interim financial report for the first quarter ‘ICorp.’ wants to defer Rs. 21 crores expenditure to third quarter on the argument that third quarter is having more sales, therefore third quarter should be debited by higher expenditure, considering the seasonal nature of business. The expenditures are uniform throughout all quarters. Calculate the result of first quarter as per AS 25 and comment on the company’s view. (c) Top & Top Limited has set up its business in a designated backward area which entitles the company to receive from the Government of India a subsidy of 20% of the cost of investment. Having fulfilled all the conditions under the scheme, the company on its investment of Rs. 50 crore in capital assets, received Rs. 10 crore from the Government in January, 2005 (accounting period being 2004-2005). The company wants to treat this receipt as an item of revenue and thereby reduce the losses on profit and loss account for the year ended 31st March, 2005. Keeping in view the relevant Accounting Standard, discuss whether this action is justified or not. ( 4 + 4 + 4 = 12 marks)(May, 2005) Answer (a) (i) Present value of residual value = Rs. 40,000  0.7513 = Rs. 30,052 Present value of lease payments = Rs. 3,00,000 – Rs. 30,052 = Rs. 2,69,948.                                                              There may be some percentage of sales given herein. Accounting Theory CA Final (Existing) 33 The present value of lease payments being 89.98%       100 3,00,000 2,69,948 of the fair value, i.e. being a substantial portion thereof, the lease constitutes a finance lease. (ii) Calculation of unearned finance income Rs. Gross investment in the lease [(Rs.1,08,552   3) + Rs. 40,000] 3,65,656 Less: Cost of the equipment 3,00,000 Unearned finance income 65,656 Note: - In the above solution, annual lease payment has been determined on the basis that the present value of lease payments plus residual value is equal to the fair value (cost) of the asset. (b) Result of the first quarter ended 31st March, 2005 (Rs. in crores) Turnover 50 Add: Other Income Nil Total 50 Less: Change in inventories Nil Salaries and other cost 30 Administrative and selling expenses (8 + 2) 10 40 Profit 10 As per AS 25 on Interim Financial Reporting, the income and expense should be recognised when they are earned and incurred respectively. As per para 38 of AS 25, the costs should be anticipated or deferred only when (i) it is appropriate to anticipate that type of cost at the end of the financial year, and (ii) costs are incurred unevenly during the financial year of an enterprise. Therefore, the argument given by I-Corp relating to deferment of Rs. 21 crores is not tenable as expenditures are uniform through out all quarters. (c) As per para 10 of AS 12 ‘Accounting for Government Grants’, where the government grants are of the nature of promoters’ contribution, i.e. they are given with reference to the total investment in an undertaking or by way of contribution towards its total capital outlay (for example, central investment subsidy scheme) and no repayment is ordinarily expected in respect thereof, the grants are treated as capital reserve which can be neither distributed as dividend nor considered as deferred income. In the given case, the subsidy received is neither in relation to specific fixed asset nor in relation to revenue. Thus it is inappropriate to recognise government grants in the profit and loss statement, since they are not earned but represent an incentive provided by government without related costs. The correct treatment is to credit the subsidy to capital reserve. Therefore, the accounting treatment followed by the company is not proper. Question 25 (a) Venus Ltd. has an asset, which is carried in the Balance Sheet on 31.3.2005 at Rs. 500 lakhs. As at that date the value in use is Rs. 400 lakhs and the net selling price is Rs. 375 lakhs. From the above data: (i) Calculate impairment loss. (ii) Prepare journal entries for adjustment of impairment loss.                                                                Annual lease payments = 1,08,552 Rs. 4868 . 2948 , 69 , 2 . Rs  (approx.) Accounting Theory CA Final (Existing) 34 (iii) Show, how impairment loss will be shown in the Balance Sheet. (b) Himalaya Ltd. in the past three years spent Rs. 75,00,000 to develop a Drug to treat Cancer, which was charged to Profit and Loss Account since they did not meet AS 8 criteria for capitalization. In the current year approval of the concerned Government Authority has been received. The Company wishes to capitalize Rs. 75,00,000 and disclose it as a prior period item. Is it correct? Give reason for your views. (c) Bottom Ltd. entered into a sale deed for its immovable property before the end of the year. But registration was done with registrar subsequent to Balance Sheet date. But before finalisation, is it possible to recognise the sale and the gain at the Balance Sheet date? Give your view with reasons. (d) ` In view of the provisions of Accounting Standard 25 on Interim Financial Reporting, on what basis will you calculate, for an interim period, the provision in respect of defined benefit schemes like pension, gratuity etc. for the employees? (6 + 5 + 5+5 = 21Marks)(Nov.2005) Answer (a) (i) Recoverable amount is higher of value in use Rs. 400 lakhs and net selling price Rs. 375 lakhs. Recoverable amount = Rs. 400 lakhs Impairment loss = Carried Amount – Recoverable amount = Rs. 500 lakhs – Rs. 400 lakhs = Rs. 100 lakhs. (ii) Journal Entries Particulars Dr. Cr. Amount Amount Rs. in lakhs Rs. in lakhs (i) Impairment loss account Dr. 100 To Asset 100 (Being the entry for accounting impairment loss) (ii) Profit and loss account Dr. 100 To Impairment loss 100 (Being the entry to transfer impairment loss to profit and loss account) (iii) Balance Sheet of Venus Ltd. as on 31.3.2005 Rs. in lakhs Asset less depreciation 500 Less: Impairment loss 100 400 (b) AS 8 stands withdrawn w.e.f. 1st April, 2003 i.e. the date from which AS 26 ‘Intangible Assets’ becomes mandatory. In any case, under either standard, the condition for recognition of a research and development asset has to be fulfilled when the expenditure was incurred. If the recognition conditions are not fulfilled the amount has to be charged to the profit and loss account. Once the amount is charged to the Profit and Loss account, such amount cannot be restated later as a Research and Development Asset when the condition for recognition get fulfilled. The Company therefore cannot capitalize Rs. 75,00,000 even as a prior period item. (c) Yes, both sales and gain of Bottom Ltd. should be recognized. In accordance with AS 9 at the Balance Sheet date and what was pending was merely a formality to register the deed. It is clear that significant risk and rewards of ownership had passed before the balance sheet date. Further the registration post the balance sheet date confirms the condition of sale at the balance sheet date as per AS 4. (d) Accounting Standard 25 suggests that provision in respect of defined benefit schemes like pension and gratuity for an interim period should be calculated based on the year-to-date basis by using the actuarially determined rates at the end of the prior financial year, adjusted for significant market fluctuations since that time and for significant curtailments, settlements or other significant one-time events. Accounting Theory CA Final (Existing) 35 Question 26 (a) In May, 2004 Speed Ltd. took a bank loan to be used specifically for the construction of a new factory building. The construction was completed in January, 2005 and the building was put to its use immediately thereafter. Interest on the actual amount used for construction of the building till its completion was Rs. 18 lakhs, whereas the total interest payable to the bank on the loan for the period till 31st March, 2005 amounted to Rs. 25 lakhs. Can Rs. 25 lakhs be treated as part of the cost of factory building and thus be capitalized on the plea that the loan was specifically taken for the construction of factory building? (b) Distinguish between “Timing differences” and “Permanent differences” referred to in AS 22 on Accounting for Taxes, giving 2 examples of each. (4 + 4 = 8Marks)( Nov. 2005) Answer (a) AS 16 clearly states that capitalization of borrowing costs should cease when substantially all the activities necessary to prepare the qualifying asset for its intended use are completed. Therefore, interest on the amount that has been used for the construction of the building upto the date of completion (January, 2005) i.e. Rs. 18 lakhs alone can be capitalized. It cannot be extended to Rs. 25 lakhs. (b) Timing differences are the differences between taxable income and accounting income for a period that originate in one period and are capable of reversal in one or more subsequent periods. Examples: (i) Unabsorbed depreciation and, carry forward of losses which can be set-off against future taxable income. (ii) Statutory dues deferred for payment under Section 43B of the Income-Tax Act. “Permanent Differences” are the differences between taxable income and accounting income for a period that originate in one period but do not reverse subsequently. Examples: (i) Agricultural income. (ii) Donations/contributions disallowed for tax purposes Question 27 (a) Global Ltd. has initiated a lease for three years in respect of an equipment costing Rs.1,50,000 with expected useful life of 4 years. The asset would revert to Global Limited under the lease agreement. The other information available in respect of lease agreement is: (i) The unguaranteed residual value of the equipment after the expiry of the lease term is estimated at Rs.20,000. (ii) The implicit rate of interest is 10%. (iii) The annual payments have been determined in such a way that the present value of the lease payment plus the residual value is equal to the cost of asset. Ascertain in the hands of Global Ltd. (i) The annual lease payment. (ii) The unearned finance income. (iii) The segregation of finance income, and also, (iv) Show how necessary items will appear in its profit and loss account and balance sheet for the various years. (8 marks) (b) Swift Ltd. acquired a patent at a cost of Rs.80,00,000 for a period of 5 years and the product life-cycle is also 5 years. The company capitalized the cost and started amortizing the asset at Rs.10,00,000 per annum. After two years it was found that the product life-cycle may continue for another 5 years from then. The net cash flows from the product during these 5 years were expected to be Rs.36,00,000,Rs.46,00,000, Rs.44,00,000, Rs.40,00,000 and Rs.34,00,000. Find out the amortization cost of the patent for each of the years. ( 4 marks ) (c) The Chief Accountant of Sports Ltd. gives the following data regarding its six segments: Rs. In lakhs Particulars M N O P Q R Total Accounting Theory CA Final (Existing) 36 Segment Assets 40 80 30 20 20 10 200 Segment Results 50 -190 10 10 -10 30 -100 Segment Revenue 300 620 80 60 80 60 1,200 The Chief accountant is of the opinion that segments “M” and “N” alone should be reported. Is he justified in his view? Discuss. . ( 4 marks ) ( May, 2006) Answer (a) (i) Calculation of Annual Lease Payment Rs. Cost of the equipment 1,50,000 Unguaranteed Residual Value 20,000 PV of residual value for 3 years @ 10% (Rs.20,000 x 0.751) 15,020 Fair value to be recovered from Lease Payment (Rs.1,50,000 – Rs.15,020) 1,34,980 PV Factor for 3 years @ 10% 2.487 Annual Lease Payment (Rs. 1,34,980 / PV Factor for 3 years @ 10% i.e. 2.487) 54,275 (ii) Unearned Financial Income Total lease payments [Rs. 54,275 x 3] 1,62,825 Add: Residual value 20,000 Gross Investments 1,82,825 Less: Present value of Investments (Rs.1,34,980 + Rs.15,020) 1,50,000 Unearned Financial Income 32,825 (iii) Segregation of Finance Income Year Lease Rentals Rs. Finance Charges @ 10% on outstanding amount of the year Rs. Repayment Rs. Outstanding Amount Rs. 0 - - - 1,50,000 I 54,275 15,000 39,275 1,10,725 II 54,275 11,073 43,202 67,523 III 74,275  6,752 67,523 -- 1,82,825 32,825 1,50,000 (iv) Profit and Loss Account ( Relevant Extracts) Credit side Rs.                                                               Annual lease payments are considered to be made at the end of each accounting year.  Rs. 74,275 includes unguaranteed residual value of equipment amounting Rs. 20,000. Accounting Theory CA Final (Existing) 37 I Year By Finance Income 15,000 II year By Finance Income 11,073 III year By Finance Income 6,752 Balance Sheet  ( Relevant Extracts) Assets side Rs. Rs. I year Lease Receivable 1,50,000 Less: Amount Received 39,275 1,10,725 II year Lease Receivable 1,10,725 Less: Received 43,202 67,523 III year :Lease Amount Receivable 67,523 Less: Amount received 47,523 Residual value 20,000 NIL Notes to Balance Sheet Year 1 Rs . Minimum Lease Payments (54,275 + 54,275) 1,08,550 Residual Value 20,000 1,28,550 Unearned Finance Income(11,073+ 6,752) 17,825 Lease Receivables 1,10,725 Classification: Not later than 1 year Later than 1 year but not more than 5 years Total 43,202 67,523 1,10,725 Year II: Minimum Lease Payments 54,275 Residual Value (Estimated) 20,000 74,275 Unearned Finance Income 6,752 Lease Receivables (not later than 1year) 67,523 III Year: Lease Receivables (including residual value) 67,523 Amount Received 67,523 NIL (b) Swift Limited amortised Rs.10,00,000 per annum for the first two years i.e. Rs.20,00,000. The remaining carrying cost can be amortized during next 5 years on the basis of net cash flows arising from the sale of the product. The amortisation may be found as follows: Year Net cash flows Rs Amortization Ratio Amortization Amount Rs. Accounting Theory CA Final (Existing) 38 I - 0.125 10,00,0001 II - 0.125 10,00,000 III 36,00,000 0.180 10,80,000 IV 46,00,000 0.230 13,80,000 V 44,00,000 0.220 13,20,000 VI 40,00,000 0.200 12,00,000 VII 34,00,000 0.170 10,20,000 Total 2,00,00,000 1.000 80,00,000 It may be seen from above that from third year onwards, the balance of carrying amount i.e., Rs.60,00,000 has been amortized in the ratio of net cash flows arising from the product of Swift Ltd. Note: The answer has been given on the basis that the patent is renewable and Swift Ltd. got it renewed after expiry of five years. (c) As per para 27 of AS 17 ‘Segment Reporting’, a business segment or geographical segment should be identified as a reportable segment if: (i) Its revenue from sales to external customers and from other transactions with other segments is 10% or more of the total revenue- external and internal of all segments; or (ii) Its segment result whether profit or loss is 10% or more of: (1) The combined result of all segments in profit; or (2) The combined result of all segments in loss, whichever is greater in absolute amount; or (iii) Its segment assets are 10% or more of the total assets of all segments. If the total external revenue attributable to reportable segments constitutes less than 75% of total enterprise revenue, additional segments should be identified as reportable segments even if they do not meet the 10% thresholds until atleast 75% of total enterprise revenue is included in reportable segments. (a) On the basis of turnover criteria segments M and N are reportable segments. (b) On the basis of the result criteria, segments M, N and R are reportable segments (since their results in absolute amount is 10% or more of Rs.200 lakhs). (c) On the basis of asset criteria, all segments except R are reportable segments. Since all the segments are covered in atleast one of the above criteria all segments have to be reported upon in accordance with Accounting Standard (AS) 17. Hence, the opinion of chief accountant is wrong. Question 28 (a) Narmada Ltd. sold goods for Rs.90 lakhs to Ganga Ltd. during financial year ended 31-3-2006. The Managing Director of Narmada Ltd. own 100% of Ganga Ltd. The sales were made to Ganga Ltd. at normal selling prices followed by Narmada Ltd. The Chief accountant of Narmada Ltd contends that these sales need not require a different treatment from the other sales made by the company and hence no disclosure is necessary as per the accounting standard. Is the Chief Accountant correct? (b) Milton Ltd. is a full tax free enterprise for the first 10 years of its existence and is in the second year of its operations. Depreciation timing difference resulting in a deferred tax liability in years 1 and 2 is Rs.200 lakhs and 400 lakhs respectively. From the 3 rd year onwards, it is expected that the timing difference would reverse each year by Rs.10 lakhs. Assuming tax rate @35%, find out the deferred tax liability at the end of the second year and any charge to the profit and loss account. (c) Victory Ltd. purchased goods on credit from Lucky Ltd. for Rs.250 crores for export. The export order was cancelled. Victory Ltd. decided to sell the same goods in the local market with a price discount. Lucky Ltd. was requested to offer a price                                                              1 It has been assumed that the company had amortized the patent at Rs. 10,00,000 per annum in the first two years on the basis of economic benefits derived from the product manufactured under the patent. Accounting Theory CA Final (Existing) 39 discount of 15%. The Chief Accountant of Lucky Ltd. wants to adjust the sales figure to the extent of the discount requested by Victory Ltd. Discuss whether this treatment is justified. (d) Accountants of Poornima Ltd. show a net profit of Rs.7,20,000 for the third quarter of 2005 after incorporating the following: (i) Bad debts of Rs.40,000 incurred during the quarter. 50% of the bad debts have been deferred to the next quarter. (ii) Extra ordinary loss of Rs.35,000 incurred during the quarter has been fully recognized in this quarter. (iii) Additional depreciation of Rs.45,000 resulting from the change in the method of charge of depreciation. Ascertain the correct quarterly income. (4x4=16 Marks)(May, 2006) Answer (a) As per paragraph 13 of AS 18 ‘Related Party Disclosures’, Enterprises over which a key management personnel is able to exercise significant influence are related parties. This includes enterprises owned by directors or major shareholders of the reporting enterprise that have a member of key management in common with the reporting enterprise. In the given case, Narmada Ltd. and Ganga Ltd are related parties and hence disclosure of transaction between them is required irrespective of whether the transaction was done at normal selling price. Hence the contention of Chief Accountant of Narmada Ltd is wrong. (b) In the case of tax free companies, no deferred tax liability is recognized, in respect of timing differences that originate and reverse in the tax holiday period. Deferred tax liability or asset is created in respect of timing differences that originate in a tax holiday period but are expected to reverse after the tax holiday period. For this purpose, adjustments are done in accordance with the FIFO method. Of Rs.200 lakhs, Rs.80 lakhs will reverse in the tax holiday period. Therefore, Deferred Tax Liability will be created on Rs.120 lakhs @ 35% (i.e.) Rs.42 lakhs. In the second year, the entire Rs.400 lakhs will reverse only after the tax holiday period. Therefore, deferred tax charge in the Profit and Loss Account will be Rs.400 x 35% = 140 lakhs and deferred tax liability in the Balance Sheet will be (42+140) = Rs.182 lakhs. (c) Lucky Ltd. had sold goods to Victory Ltd on credit worth for Rs.250 crores and the sale was completed in all respects. Victory Ltd’s decision to sell the same in the domestic market at a discount does not affect the amount recorded as sales by Lucky Ltd. The price discount of 15% offered by Lucky Ltd. after request of Victory Ltd. was not in the nature of a discount given during the ordinary course of trade because otherwise the same would have been given at the time of sale itself. Now, as far Lucky Ltd is concerned, there appears to be an uncertainty relating to the collectability of the debt, which has arisen subsequent to the time of sale therefore, it would be appropriate to make a separate provision to reflect the uncertainty relating to collectability rather than to adjust the amount of revenue originally recorded. Therefore, such discount should be written off to the profit and loss account and not shown as deduction from the sales figure. (d) In the above case, the quarterly income has not been correctly stated. As per AS 25 “Interim Financial Reporting”, the quarterly income should be adjusted and restated as follows: Bad debts of Rs. 40,000 have been incurred during current quarter. Out of this, the company has deferred 50% (i.e.) Rs. 20,000 to the next quarter. Therefore, Rs. 20,000 should be deducted from Rs. 7,20,000. The treatment of extra-ordinary loss of Rs. 35,000/- being recognized in the same quarter is correct. Recognising additional depreciation of Rs. 45,000 in the same quarter is in tune with AS 25 .Hence, no adjustments are required for these two items. Poornima Ltd should report quarterly income as Rs.7,00,000 (Rs. 7,20,000–Rs. 20,000). Question 29 (a) A company had imported raw materials worth US Dollars 6,00,000 on 5 th January, 2005, when the exchange rate was Rs.43 per US Dollar. The company had recorded the transaction in the books at the above mentioned rate. The payment for the import transaction was made on 5 th April, 2005 when the exchange rate was Rs.47 per US Dollar. However, on 31st March, 2005, the rate of exchange was Rs.48 per US Dollar. The company passed an entry on 31st March, 2005 adjusting the cost of raw materials consumed for the difference between Rs.47 and Rs.43 per US Dollar. In the background of the relevant accounting standard, is the company’s accounting treatment correct? Discuss. Accounting Theory CA Final (Existing) 40 (b) A private limited company manufacturing fancy terry towels had valued its closing stock of inventories of finished goods at the realisable value, inclusive of profit and the export cash incentives. Firm contracts had been received and goods were packed for export, but the ownership in these goods had not been transferred to the foreign buyers. Comment on the valuation of the stocks by the company. (c) A company with a turnover of Rs.250 crores and an annual advertising budget of Rs.2 crore had taken up the marketing of a new product. It was estimated that the company would have a turnover of Rs. 25 crores from the new product. The company had debited to its Profit and Loss account the total expenditure of Rs.2 crore incurred on extensive special initial advertisement campaign for the new product. Is the procedure adopted by the company correct? (d) A company deals in petroleum products. The sale price of petrol is fixed by the government. After the Balance Sheet date, but before the finalisation of the company’s accounts, the government unexpectedly increased the price retrospectively. Can the company account for additional revenue at the close of the year? Discuss. (e) Mohur Ltd. has equity capital of Rs.40,00,000 consisting of fully paid equity shares of Rs.10 each. The net profit for the year 2004-05 was Rs.60,00,000. It has also issued 36,000, 10% convertible debentures of Rs.50 each. Each debenture is convertible into five equity shares. The tax rate applicable is 30%. Compute the diluted earnings. (4 Marks each)(Nov. 2006) Answer (a) As per AS 11 (revised 2003), ‘The Effects of Changes in Foreign Exchange Rates’, monetary items denominated in a foreign currency should be reported using the closing rate at each balance sheet date. The effect of exchange difference should be taken into profit and loss account. Sundry creditors is a monetary item, hence should be valued at the closing rate i.e, Rs.48 at 31 st March, 2005 irrespective of the payment for the same subsequently at lower rate in the next financial year. The difference of Rs.5 (48-43) per US dollar should be shown as an exchange loss in the profit and loss account for the year ended 31 st March, 2005 and is not to be adjusted against the cost of raw- materials. In the subsequent year, the company would record an exchange gain of Re.1 per US dollar, i.e., the difference between Rs.48 and Rs.47 per Us dollar. Hence, the accounting treatment adopted by the company is incorrect. (b) Accounting Standard 2 “Valuation of Inventories” states that inventories should be valued at lower of historical cost and net realisable value. AS 9 on “Revenue Recognition” states, “at certain stages in specific industries, such as when agricultural crops have been harvested or mineral ores have been extracted, performance may be substantially complete prior to the execution of the transaction generating revenue. In such cases, when sale is assured under forward contract or a government guarantee or when market exists and there is a negligible risk of failure to sell, the goods invoiced are often valued at Net-realisable value.” Terry Towels do not fall in the category of agricultural crops or mineral ores. Accordingly, taking into account the facts stated, the closing stock of finished goods (Fancy terry towel) should have been valued at lower of cost and net- realisable value and not at net realisable value. Further, export incentives are recorded only in the year the export sale takes place. Therefore, the policy adopted by the company for valuing its closing stock of inventories of finished goods is not correct. (c) According to paras 55 and 56 of AS 26 ‘Intangible Assets’, “expenditure on an intangible item should be recognised as an expense when it is incurret unless it forms part of the cost of an intangible asset”. In the given case, advertisement expenditure of Rs. 2 crores had been taken up for the marketing of a new product which may provide future economic benefits to an enterprise by having a turnover of Rs.25 crores. Here, no intangible asset or other asset is acquired or created that can be recognised. Therefore, the accounting treatment by the company of debiting the entire advertising expenditure of Rs.2 crores to the Profit and Loss account of the year is correct. (d) According to para 8 of AS 4 (Revised 1995), the unexpected increase in sale price of petrol by the government after the balance sheet date cannot be regarded as an event occurring after the Balance Sheet date, which requires an adjustment at the Balance Sheet date, since it does not represent a condition present at the balance sheet date. The revenue should be recognized only in the subsequent year with proper disclosures. The retrospective increase in the petrol price should not be considered as a prior period item, as per AS 5, because there was no error in the preparation of previous period’s financial statements. (e) Interest on Debentures @ 10% for the year 36,00050 10010 Accounting Theory CA Final (Existing) 41 = Rs.1,80,000 Tax on interest @ 30% = Rs.54,000 Diluted Earnings (Adjusted net profit) = (60,00,000 + 1,80,000-54,000) = Rs. 61,26,000 Question 30 (a) During the course of the last three years, a company owning and operating Helicopters lost four Helicopters. The company Accountant felt that after the crash, the maintenance provision created in respect of the respective helicopters was no longer required, and proposed to write back to the Profit and Loss account as a prior period item. Is the Company’s proposed accounting treatment correct? Discuss. (b) Mr. ‘X’ as a contractor has just entered into a contract with a local municipal body for building a flyover. As per the contract terms, ‘X’ will receive an additional Rs.2 crore if the construction of the flyover were to be finished within a period of two years of the commencement of the contract. Mr. X wants to recognize this revenue since in the past he has been able to meet similar targets very easily. Is X correct in his proposal? Discuss. (c) A Company is in the process of setting up a production line for manufacturing a new product. Based on trial runs conducted by the company, it was noticed that the production lines output was not of the desired quality. However, company has taken a decision to manufacture and sell the sub-standard product over the next one year due to the huge investment involved. In the background of the relevant accounting standard, advise the company on the cut-off date for capitalization of the project cost. (d) A Company has an inter-segment transfer pricing policy of charging at cost less 10%. The market prices are generally 25% above cost. Is the policy adopted by the company correct? (4+4+4+4 = 16 Marks)(May, 2007) Answer (a) The balance amount of maintenance provision written back to profit and loss account, no longer required due to crash of the helicopters, is not a prior period item because there was no error in the preparation of previous periods’ financial statements. The term ‘prior period items’, as defined in AS 5 (revised) “Net Profit or Loss for the Period, Prior Period Items and Changes In Accounting Policies”, refer only to income or expenses which arise in the current period as a result of errors or omissions in the preparation of the financial statements of one or more prior periods. As per paragraph 8 of AS 5, extraordinary items should be disclosed in the statement of profit and loss as a part of net profit or loss for the period. The nature and the amount of each extraordinary item should be separately disclosed in the statement of profit and loss in a manner that its impact on current profit or loss can be perceived. The amount so written-back (If material) should be disclosed as an extraordinary item as per AS 5. (b) According to para 14 of AS 7 (Revised) ‘Construction Contracts’, incentive payments are additional amounts payable to the contractor if specified performance standards are met or exceeded. For example, a contract may allow for an incentive payment to the contractor for early completion of the contract. Incentive payments are included in contract revenue when: (i) the contract is sufficiently advanced that it is probable that the specified performance standards will be met or exceeded; and (ii) the amount of the incentive payment can be measured reliably. In the given problem, the contract has not even begun and hence the contractor (Mr. X) should not recognize any revenue of this contract. (c) As per provisions of AS 10 ‘Accounting for Fixed Assets’, expenditure incurred on start-up and commissioning of the project, including the expenditure incurred on test runs and experimental production, is usually capitalized as an indirect element of the construction cost. However, the expenditure incurred after the plant has begun commercial production i.e., production intended for sale or captive consumption, is not capitalized and is treated as revenue expenditure even though the contract may stipulate that the plant will not be finally taken over until after the satisfactory completion of the guarantee period. In the present case, the company did stop production even if the output was not of the desired quality, and continued the sub-standard production due to huge investment involved in the project. Capitalization should cease at the end of the trial run, since the cut-off date would be the date when the trial run was completed. (d) AS 17 ‘Segment Reporting’ requires that inter-segment transfers should be measured on the basis that the enterprise actually used to price these transfers. The basis of pricing inter-segment transfers and any change therein should be disclosed in the financial statements. Hence, the enterprise can have its own policy for pricing inter-segment transfers and hence, inter-segment transfers may be based on cost, below cost or market price. However, whichever policy is Accounting Theory CA Final (Existing) 42 followed, the same should be disclosed and applied consistently. Therefore, in the given case inter-segment transfer pricing policy adopted by the company is correct if, followed consistently. Question 31 Write short notes on: (a) Impairment of asset and its application to inventory. (b) Treatment of borrowing costs. (c) Accounting for investment by a holding company in subsidiaries. (4 marks each) (May, 2007) (d) Concept of Materiality. (6 Marks) (Nov. 2007) Answer (a) The objective of AS 28 ‘Impairment of Assets’ is to prescribe the procedures that an enterprise applies to ensure that its assets are carried at no more than their recoverable amount. An asset is carried at more than its recoverable amount if its carrying amount exceeds the amount to be recovered through use or sale of the asset. If this is the case, the asset is described as impaired and this Statement requires the enterprise to recognize an impairment loss. This standard should be applied in accounting for the impairment of all assets, other than (i) inventories (AS 2, Valuation of Inventories); (ii) assets arising from construction contracts (AS 7, Accounting for Construction Contracts); (iii) financial assets, including investments that are included in the scope of AS 13, Accounting for Investments; and (iv) deferred tax assets (AS 22, Accounting for Taxes on Income). AS 28 does not apply to inventories, assets arising from construction contracts, deferred tax assets or investments because other accounting standards applicable to these assets already contain specific requirements for recognizing and measuring the impairment related to these assets. (b) According to AS 16, borrowing costs are interest and other costs incurred by an enterprise in connection with the borrowing of funds. Borrowing costs may include: (i) interest and commitment charges on bank borrowings and other short-term and long-term borrowings; (ii) amortization of discounts or premiums relating to borrowings; (iii) amortization of ancillary costs incurred in connection with the arrangement of borrowings; (iv) finance charges in respect of assets acquired under finance leases or under other similar arrangements; and (v) exchange differences arising from foreign currency borrowings to the extent that they are regarded as an adjustment to interest costs. Borrowing costs that are directly attributable to the acquisition, construction or production of a qualifying asset  should be capitalized as part of the cost of that asset. Other borrowing costs should be recognized as an expense in the period in which they are incurred. The capitalization of borrowing costs as part of the cost of a qualifying asset should commence when the conditions specified in AS 16 are satisfied. (c) Investments by a holding company in the shares of its subsidiary company are normally considered as long term investments. Indian holding companies show investment in subsidiary just like any other investment and generally classify it as trade investment. As per AS 13 ‘Accounting for Investments’, investments are classified as long term and current investments. A current investment is an investment that by its nature is readily realizable and is intended to be held for more than one year from the date of acquisition. A long term investment is one that is not a current one. Costs of investment include besides acquisition charges, expenses such as brokerage, fees and duties. If an investment is acquired wholly or partly by an issue of shares or other securities, the acquisition cost is determined by taking the fair value of the shares/securities issued. If an investment were to be acquired in exchange – part or whole – for another asset, the acquisition cost of the investment is determined with reference to the value of the other asset exchanged. Dividends received out of incomes earned by a subsidiary before the acquisition of the shares by the holding company and not treated as income but treated as recovery of cost of the assets (investment made in the subsidiary). The carrying cost for current investment is the lower of cost or fair/market value whereas investment in the shares of the subsidiary (treated as long term) is carried normally at cost. (d) Para 17 of AS 1 ‘Disclosure of Accounting Policies’, states that financial statements should disclose all material items, i.e., items the knowledge of which might influence the decisions of the user of the financial statements. Materiality depends on the size of item or error judged in the particular circumstances of its omission or misstatement. From a positive perspective, materiality has to do with the significance of an item or event to warrant attention in the accounting process. From a negative view point, materiality is critical because otherwise a great deal of time might be spent on trivial matters in the accounting process. Individual judgements are required to assess materiality, or to decide what the appropriate minimum quantitative criteria are to be set for given situations. What is material to one organisation, may not be material for another organisation.                                                              A qualifying asset is an asset that necessarily takes a substantial period of time1 to get ready for its intended use or sale. Accounting Theory CA Final (Existing) 43 The relevance of information is affected by its materiality. Information is material if its misstatements (i.e., omission or erroneous statement) could influence the economic decisions of users taken on the basis of the financial information. Materiality provides a threshold or cut-off point rather than being a primary qualitative characteristic which the information must have if it is to be useful. Question 32 (a) Arrange and redraft the following Cash Flow Statement in proper order keeping in mind the requirements of AS 3: Rs. (in lacs) Rs.(in lacs) Net Profit 60,000 Add: Sale of Investments 70,000 Depreciation on Assets 11,000 Issue of Preference Shares 9,000 Loan raised 4,500 Decrease in Stock 12,000 1,66,500 Less: Purchase of Fixed Assets 65,000 Decrease in Creditors 6,000 Increase in Debtors 8,000 Exchange gain 8,000 Profit on sale of investments 12,000 Redemption of Debenture 5,700 Dividend paid 1,400 Interest paid 945 1,07,045 59,455 Add: Opening cash and cash equivalent 12,341 Closing cash and cash equivalent 71,796 (b) P Ltd. has 60% voting right in Q Ltd. Q Ltd. has 20% voting right in R Ltd. Also, P Ltd. directly enjoys voting right of 14% in R Ltd. R Ltd. is a listed company and regularly supplies goods to P Ltd. The management of R Ltd. has not disclosed its relationship with P Ltd. How would you assess the situation from the viewpoint of AS 18 on Related Party Disclosures? (c) Lessee Ltd. took a machine on lease from Lessor Ltd., the fair value being Rs.7,00,000. The economic life of the machine as well as the lease term is 3 years. At the end of each year Lessee Ltd. pays Rs.3,00,000. Guaranteed Residual Value (GRV) is Rs.22,000 on expiry of the lease. Implicit Rate of Return (IRR) is 15% p.a. and present value factors at 15% are 0.869, 0.756 and 0.657 at the end of first, second and third years respectively. Calculate the value of machine to be considered b y Lessee Ltd. and the interest (Finance charges) in each year. (6+4+6 = 16 Marks)(Nov. 2007) Answer (a) Cash Flow Statement Cash flows from operating activities (Rs. in lacs) Net profit 60,000 Less: Exchange gain (8,000) Less: Profit on sale of investments (12,000) 40,000 Add: Depreciation on assets 11,000 Accounting Theory CA Final (Existing) 44 Change in current assets and current liabilities 51,000 (-) Increase in debtors (8,000) (+) Decrease in stock 12,000 (-) Decrease in creditors (6,000) (2,000) Net cash from operating activities 49,000 Cash flows from investing activities Sale of investments 70,000 Purchase of fixed assets (65,000) Net cash from Investing activities 5,000 Cash flows from financing activities Issue of preference shares 9,000 Loan raised 4,500 Redemption of Debentures (5,700) Interest paid (945) Dividend paid (1,400) Net cash from financing activities 5,455 Net increase in cash & cash equivalents 59,455 Add: Opening cash and cash equivalents 12,341 Closing cash and cash equivalents 71,796 (b) P Ltd. has direct economic interest in R Ltd to the extent of 14%, and through Q Ltd. in which it is the majority shareholders, it has further control of 12% in R Ltd. (60% of Q Ltd’s 20%). These two taken together (14% + 12%) make the total control of 26%. Para 10 of AS 18 ‘Related Party Disclosures’, defines related party as one that has at any time during the reporting period, the ability to control the other party or exercise significant influence over the other party in making financial and/or operating decisions. Here, Control is defined as ownership directly or indirectly of more than one-half of the voting power of an enterprise; and Significant Influence is defined as participation in the financial and/or operating policy decisions of an enterprise but not control of those policies. In the present case, control of P Ltd. in R Ltd. directly and through Q Ltd., does not go beyond 26%. However, as per para 12 of AS 18, significant influence may be exercised as an investing party (P Ltd.) holds, directly or indirectly through intermediaries 20% or more of the voting power of the R Ltd. As R Ltd. is a listed company and regularly supplies goods to P Ltd. therefore, related party disclosure, as per AS 18, is required. (c) Value of machine will be lower of the fair value or present value (PV) of Minimum Lease Payments (MLP). Present value (PV) of Minimum Lease Payments (MLP) Year MLP PV at 15% PV Amount Rs. Rs. 1 3,00,000 0.869 2,60,700 2 3,00,000 0.756 2,26,800 3 3,22,000 (considering residual value) 0.657 2,11,554 6,99,054 Since PV of MLP Rs. 6,99,054 being lower than the fair value Rs. 7,00,000, therefore, value of machine will be taken as Rs.6,99,054. Calculation of interest (finance charges) Year Liability Interest at 15% Principal Lease rental Accounting Theory CA Final (Existing) 45 Rs. Rs. Rs. Rs. 6,99,054 1,04,858 1,95,142 3,00,000 1st Less: Principal 1,95,142 (Rental – Interest) 5,03,912 75,587 2,24,413 3,00,000 2 nd Less: Principal 2,24,413 (Rental – Interest) 2,79,499 41,925 2,58,075 3,00,000 3 rd Less: Principal 2,58,075 (Rental – Interest) Residual value 21,424 Question 33 X Ltd. began construction of a new building on 1 st January, 2007. It obtained Rs.1 lakh special loan to finance the construction of the building on 1st January, 2007 at an interest rate of 10%. The company’s other outstanding two non- specific loans were: Amount Rate of Interest Rs.5,00,000 11% Rs.9,00,000 13% The expenditure that were made on the building project were as follows: Rs. January 2007 2,00,000 April 2007 2,50,000 July 2007 4,50,000 December 2007 1,20,000 Building was completed by 31 st December, 2007. Following the principles prescribed in AS-16 ‘Borrowing Cost,’ calculate the amount of interest to be capitalized and pass one Journal Entry for capitalizing the cost and borrowing cost in respect of the building. (10 Marks) (May, 2008) Answer (i) Computation of average accumulated expenses Rs. Rs. 2,00,000 x 12 / 12 = 2,00,000 Rs. 2,50,000 x 9 / 12 = 1,87,500 Rs. 4,50,000 x 6 / 12 = 2,25,000 Rs. 1,20,000 x 1 / 12 = 10,000 6,22,500 (ii) Calculation of average interest rate other than for specific borrowings Amount of loan (in Rs.) Rate of interest Ampount of interest (in Rs.) 5,00,000 11% = 55,000 9,00,000 13% = 1,17,000 14,00,000 1,72,000 Weighted average rate of interest ( 100 000 , 00 , 14000 , 72 , 1 ) = 12.285% (approx) (iii) Interest on average accumulated expenses                                                               The difference between this figure and guaranteed residual value (Rs. 22,000) is due to approximation in computing the interest rate implicit in the lease. Accounting Theory CA Final (Existing) 46 Rs. Specific borrowings (Rs. 1,00,000 X 10%) = 10,000 Non-specific borrowings (Rs. 5,22,500  X 12.285%) = 64,189 Amount of interest to be capitalized = 74,189 (iv) Total expenses to be capitalized for building Rs. Cost of building Rs.(2,00,000 + 2,50,000 + 4,50,000 + 1,20,000) 10,20,000 Add: Amount of interest to be capitalised 74,189 10,94,189 (v) Journal Entry Date Particulars Dr. (Rs.) Cr. (Rs.) 31.12.2007 Building account Dr. 10,94,189 To Bank account 10,94,189 (Being amount of cost of building and borrowing cost thereon capitalized) Question 34 (a) U.K. International Ltd. is developing a new production process. During the financial year ending 31 st March, 2007, the total expenditure incurred was Rs.50 lakhs. This process met the criteria for recognition as an intangible asset on 1st December, 2006. Expenditure incurred till this date was Rs.22 lakhs. Further expenditure incurred on the process for the financial year ending 31 st March, 2008 was Rs.80 lakhs. As at 31st March, 2008, the recoverable amount of know- how embodied in the process is estimated to be Rs.72 lakhs. This includes estimates of future cash outflows as well as inflows. You are required to calculate: (i) Amount to be charged to Profit and Loss A/c for the year ending 31 st March, 2007 and carrying value of intangible as on that date. (ii) Amount to be charged to Profit and Loss A/c and carrying value of intangible as on 31 st March, 2008. Ignore depreciation. (b) Mini Ltd. took a factory premises on lease on 1.4.07 for Rs.2,00,000 per month. The lease is operating lease. During March, 2008, Mini Ltd. relocates its operation to a new factory building. The lease on the old factory premises continues to be live upto 31.12.2010. The lease cannot be cancelled and cannot be sub-let to another user. The auditor insists that lease rent of balance 33 months upto 31.12.2010 should be provided in the accounts for the year ending 31.3.2008. Mini Ltd. seeks your advice. (c) A Cosmetic articles producing company provides the following information: Cold Cream Vanishing Cream January, 2006 – September, 2006 per month 2,00,000 2,00,000 October, 2006 – December, 2006 per month 1,00,000 3,00,000 January, 2007- March, 2007 per month 0 4,00,000 The company has enforced a gradual change in product-line on the basis of an overall plan. The Board of Directors of the company has passed a resolution in March, 2006 to this effect. The company follows calendar year as its accounting year. Should this be treated as a discontinuing operation? Give reasons in support of your answer. (5+5+5 =15 Marks) (May, 2008) Answer (a) As per AS 26 ‘Intangible Assets’                                                               (Rs. 6,22,500 – Rs. 1,00,000) Accounting Theory CA Final (Existing) 47 (i) For the year ending 31.03.2007 (1) Carrying value of intangible as on 31.03.2007: At the end of financial year 31 st March 2007, the production process will be recognized (i.e. carrying amount) as an intangible asset at a cost of Rs. 28 lakhs (expenditure incurred since the date the recognition criteria were met, i.e., on 1st December 2006). (2) Expenditure to be charged to Profit and Loss account: The Rs. 22 lakhs is recognized as an expense because the recognition criteria were not met until 1 st December 2007. This expenditure will not form part of the cost of the production process recognized in the balance sheet. (ii) For the year ending 31.03.2008 (1) Expenditure to be charged to Profit and Loss account: (Rs. in lakhs) Carrying Amount as on 31.03.2007 28 Expenditure during 2007 – 2008 80 Total book cost 108 Recoverable Amount 72 Impairment loss 36 Rs. 36 lakhs to be charged to Profit and loss account for the year ending 31.03.2008. (2) Carrying value of intangible as on 31.03.2008: (Rs. in lakhs) Total Book Cost 108 Less: Impairment loss 36 Carrying amount as on 31.03.2008 72 (b) In accordance with AS 29 ‘Provisions, Contingent Liabilities and Contingent Assets’ and ASI 30 ‘Applicability of AS 29 to Onerous Contracts’, if an enterprise has a contract that is onerous, the present obligation under the contract should be recognized and measured as a provision. In the given case, the operating lease contract has become onerous  as the economic benefit of lease contract for next 33 months up to 31.12.2010 will be nil. However, the lessee, Mini Ltd., has to pay lease rent of Rs. 66,00,000 (i.e.2,00,000 p.m. for next 33 months). Therefore, provision on account of Rs.66,00,000 is to be provided in the accounts for the year ending 31.03.08. Hence auditor is right. (c) In response to the market forces, business enterprises often abandon products or even product lines and reduce the size of their work-force. These actions are not in themselves discontinuing operations unless they satisfy the definition criteria. In the instant case the company has been gradually reducing operation in the product line of cold creams, simultaneously increasing operation in the product line of vanishing creams. The company was not disposing of any of its components. Phasing out a product line as undertaken by the company does not meet definition criteria in paragraph 3 of AS 24, namely, disposing of substantially in its entirety a component of the enterprise. Therefore, this change over is not a discontinuing operation. Question 35 From the following information of Beta Ltd. calculate Earning Per Share (EPS) in accordance with AS-20: (Rs.) Year 31.3.08 Year 31.3.07 1. Net profit before tax 3,00,000 1,00,000                                                               For a contract to qualify as an onerous contract, the unavoidable costs of meeting the obligation under the contract should exceed the economic benefits expected to be received under it. Accounting Theory CA Final (Existing) 48 2. Current tax 40,000 30,000 Tax relating to earlier years 24,000 (13,000) Deferred tax 30,000 10,000 3. Profit after tax 2,06,000 73,000 4. Other information: (a) Profit includes compensation from Central Government towards loss on account of earthquake in 2005 (non- taxable) 1,00,000 NIL (b) Outstanding convertible 6% Preference shares 1,000 issued and paid on 30.9.2006. Face value Rs.100, Conversion ratio 15 equity shares for every preference share. (c) 15% convertible debentures of Rs.1,000 each total face value Rs.1,00,000 to be converted into 10 Equity shares per debenture issued and paid on 30.6.2006. (d) Total no. of Equity shares outstanding as on 31.3.2008, 20,000 including 10,000 bonus shares issued on 1.1.08, face value Rs.100. Administrative and Collection costs 25,000 (8 Marks) (Nov. 2008) Answer (a) Calculation of Earning Per Share (EPS) of Beta Ltd. Rs. Rs. Year ended 31.3.08 Year ended 31.3.07 1. A Earning after extra ordinary items 2,00,000 70,000 (2,06,000 – 6,000) (73,000 – 3,000) B. No. of Equity Shares 20,000 20,000 C. Basic Earnings Per share [A/B] 10.00 3.50 A. Earning before extra ordinary items 1,00,000 70,000 B. No. of Equity Shares 20,000 20,000   C. Basic Earnings Per share [A/B] 5.00 3.50 2. Tax Rate applicable    40,000 + 30,000/2,00,000 35%   30,000 + 10,000/1,00,000  40% 3. A. Dividend on Weighted AveragePreference Shares 6,000 3,000 B. Incremental shares 15,000 7,500  C. EPS on Incremental Shares [A/B] 0.40 0.40 (dilutive) (dilutive) 4. Convertible Debentures    A. Increase in earnings                                                                Since the bonus issue is without consideration, the issue is treated as if it had occurred prior to the beginning of the year 2007. Accounting Theory CA Final (Existing) 49 (1,00,000 ) 65 . 10015   9,750 129 60 . 10015 000 , 00 , 1   6,750 B. Increase in shares 1,000 750 C. Increase in EPS [A/B] 9.75 9.00 (Anti dilutive) (Anti dilutive) It is anti-dilutive as it increases the EPS from continuing ordinary operations (Para 39, AS 20) Calculation of Diluted EPS Year ended 31.3.08 Rs. Year ended 31.3.07 Rs. A. Profit from continuing ordinary activities before Preference Dividend 1,06,000 73,000 No. of ordinary equity shares 20,000 20,000 Adjustment for dilutive potential of 6% convertible pref. shares 15,000 7,500 B. Total no. of shares 35,000 27,500 C. Diluted EPS from continuing ordinary operations [A/B] 3.02 2.65 D. Profit including extra ordinary items 2,06,000 73,000 E. Adjusted No. of shares 35,000 27,500 F. Diluted EPS including extra ordinary items [D/E] 5.88 2.65 Disclosure of EPS in accordance with AS 20 in the Profit and Loss Account Earning per share (Face value Rs.100) 31.3.08 (Rs. ) 31.3.07 (Rs.) Basic EPS from continuing ordinary operations 5.00 3.50 Diluted EPS from continuing ordinary operations 3.02 2.65 Question 36 (a) Golden Eagle Ltd., has been successful jewellers for the past 100 years and sales are against cash only. The company diversified into apparels. A young senior executive was put in charge of Apparels business and sales increased 5 times. One of the conditions for sales that dealers can return the unsold stocks within one month of the end of season. Sales return for the year was 25% of sales. Suggest a suitable Revenue Recognition Policy with references to AS-9. (b) Discuss the concept of Cost v/s Fair value with reference to Indian Accounting Standards. (c) A company has a scheme for payment of settlement allowance to retiring employees. Under the scheme, retiring employees are entitled to reimbursement of certain travel expenses for class they are entitled to as per company rule and to a lump-sum payment to cover expenses on food and stay during the travel. Alternatively employees can claim a lump sum amount equal to one month pay last drawn. The company’s contentions in this matter are: (i) Settlement allowance does not depend upon the length of service of employee. It is restricted to employee’s eligibility under the Travel rule of the company or where option for lump-sum payment is exercised, equal to the last pay drawn. (ii) Since it is not related to the length of service of the employees, it is accounted for on claim basis . State whether the contentions of the company are correct as per relevant Accounting Standard. Give reasons in support of your answer. (4 marks each) (Nov. 2008) Answer Accounting Theory CA Final (Existing) 50 (a) As per AS 9 “Revenue recognition”, revenue recognition is mainly concerned with the timing of recognition of revenue in statement of profit and loss of an enterprise. The amount of revenue arising on a transaction is usually determined by the agreement between the parties involved in the transaction. When uncertainties exist regarding the determination of the amount, or its associated costs, these uncertainties may influence the timing of revenue recognition. In the case of the Jewellery Business the company is selling for cash and returns are negligible. Hence, revenue can be recognized on sales. On the other hand, in Apparels Industry, the dealers have a right to return the unsold goods within one month of the end of the season. In this case, the company is bearing the risk of sales return and therefore, the company should not recognize the revenue to the extent of 25% of its sales. The company may disclose suitable revenue recognition policy in its financial statements separately for both Jewellery and Apparels business. (b) Cost vs. Fair value Cost basis: The term cost refers to cost of purchase, costs of conversion on other costs incurred in bringing the goods to its present condition and location. Assets are recorded at the amount of cash or cash equivalents paid or the fair value of the other consideration given to acquire them at the time of their acquisition. Liabilities are recorded at the amount of proceeds received in exchange for the obligation, or in some circumstances (for example, income taxes), at the amounts of cash or cash equivalents expected to be paid to satisfy the liability in the normal course of business. Fair value: Fair value of an asset is the amount at which an enterprise expects to exchange an asset between knowledgeable and willing parties in an arm’s length transaction. Indian Accounting Standards are generally based on historical cost with a very few exceptions: AS 2 “Valuation of Inventories” – Inventories are valued at net realizable value (NRV) if cost of inventories is more than NRV. AS 10 “Accounting for Fixed Assets” – Items of fixed assets that have been retired from active use and are held for disposal are stated at net realizable value if their net book value is more than NRV. AS 13 “Accounting for Investments” – Current investments are carried at lower of cost and fair value. The carrying amount of long term investments is reduced to recognise the permanent decline in value. AS 15 “Employee Benefits” – The provision for defined benefits is made at fair value of the obligations. AS 26 “Intangible Assets” – If an intangible asset is acquired in exchange for shares or other securities of the reporting enterprise, the asset is recorded at its fair value, or the fair value of the securities issued, whichever is more clearly evident. AS 28 “Impairment of Assets”– Provision is made for impairment of assets. On the other hand IFRS and US GAAPs are more towards fair value. Fair value concept requires a lot of estimation and to the extent, it is subjective in nature. (c) The present case falls under the category of defined benefit scheme under Para 49 of AS 15 (Revised) “Employee Benefits”. The said para encompasses cases where payment promised to be made to an employee at or near retirement presents significant difficulties in the determination of periodic charge to the statement of profit and loss. The contention of the Company that the settlement allowance will be accounted for on claim basis is not correct even if company’s obligation under the scheme is uncertain and requires estimation. In estimating the obligation, assumptions may need to be made regarding future conditions and events, which are largely outside the company’s control. Thus, (1) Settlement allowance payable by the company is a defined retirement benefit, covered by AS I5 (Revised). (2) A provision should be made every year in the accounts for the accruing liability on account of settlement allowance. The amount of provision should be calculated according to actuarial valuation. (3) Where, however, the amount of provision so determined is not material, the company can follow some other method of accounting for settlement allowances. Question 37 (a) XYZ Ltd., with a turnover of Rs.35 lakhs and borrowings of Rs.10 lakhs during any time in the previous year, wants to avail the exemptions available in adoption of Accounting Standards applicable to companies for the year ended 31.3.2008. Advise the management the exemptions that are available as per Companies (AS) Rules, 2006. If XYZ is a partnership firm is there any other exemptions additionally available. (b) Write short note on NACAS. (8 + 4 = 12marks)(Nov. 2008) Answer Accounting Theory CA Final (Existing) 51 (a) XYZ Ltd. is a small and medium sized enterprise (SME) company as per Companies (AS) Rules, 2006. The following relaxations and exemptions are available. 1. AS 3 “Cash Flow Statements” is not mandatory. 2. AS 17 “Segment Reporting” is not mandatory. 3. SMEs are exempt from some paragraphs of AS 19 “Leases”. 4. SMEs are exempt from disclosures of diluted EPS (both including and excluding extraordinary items). 5. SMEs are allowed to measure the ‘value in use’ on the basis of reasonable estimate thereof instead of computing the value in use by present value technique under AS 28 “Impairment of Assets”. 6. SMEs are exempt from disclosure requirements of paragraphs 66 and 67of AS 29 “Provisions , Contingent Liabilities and Contingent Assets”. 7. SMEs are exempt from certain requirements of AS 15 “Employee Benefits”. 8. Accounting Standards 21, 23, 27 are not applicable to SMEs. If XYZ is not a company, it will be treated as a level III enterprise instead of level II enterprise; XYZ Ltd. will be exempt from requirements of AS 18 “Related Party Disclosures” and AS 24 “Discontinuing Operations”. ( b) NACAS: Under Section 210 A of the Companies Act 1956, the Central Government, by notification, has constituted a committee to advise the Central Government on the formulation of accounting policies and accounting standards for adoption by companies or class of companies specified under the Act. Based on the recommendations of NACAS, the Central Government has notified AS 1 to AS 7 and AS 9 to AS 29 in Dec. 2006 in the form of Companies (Accounting Standards) Rules, 2006. Question 38 (a) Summarise the recommendations of the Institute of Chartered Accountants of India regarding accounting treatment of excise duty. (10 marks) (b) Write a short note on accounting of income during construction period. (5 marks) (c) Advise the company on the following item while from the view point of finalisation of accounts : While executing a new project, the company had to pay Rs. 50 lakhs to the State Government as part of the cost of roads built by the State Government in the vicinity of the project for the purpose of carrying machinery and materials to the project site. The road so built is the property of the State Government. (3 marks) (Nov, 1996) Answer (a) As per the Guidance Note on Accounting Treatment for Excise Duty issued by the Institute, the summary of recommendations is given as below: (i) Excise duty should be considered as a manufacturing expense and like other manufacturing expense be considered as an element of cost for inventory valuation. (ii) Where excise duty is paid on excisable goods and such goods are subsequently utilised in the manufacturing process the duty paid on such goods becomes a manufacturing cost and must be included in the valuation of work-in-progress or finished goods arising from the subsequent processing of such goods. (iii) Where the liability for excise duty has been incurred but its collection is deferred, provision for the unpaid liability should be made. The estimate of such liability can be made at the rates in force on the balance sheet date. If provision is not made, the liability should be quantified and the fact about the non-provision of such liability should be disclosed in the accounts. (iv) The excise duty cannot be treated a s a period cost. (v) If the method of accounting for excise duty or the method of inventory valuation is not in accordance with the principles explained in this guidance note, the auditor should qualify his report. Note: The ICAI has issued a separate Guidance Note on Accounting Treatment for MODVAT which sets out principles of accounting for MODVAT (now renamed as CENVAT). The Guidance Note ‘Accounting Treatment for MODVAT/CENVAT’ is, currently under revision. (b) The treatment and accounting of, income during the construction or pre-production period has been explained in para 8.1 of the Guidance Note on Treatment of Expenditure During Construction Period. According to it, it is possible that a Accounting Theory CA Final (Existing) 52 new project may earn some income from miscellaneous sources during its construction or pre-production period. Such income may be earned by way of interest from the temporary investment of surplus funds prior to their utilisation for capital or other expenditure or from sale of products manufactured during the period of test runs and experimental production. Such items of income should be disclosed separately either in the profit and loss account, where this account is prepared during construction period, or in the account/statement prepared in lieu of the profit and loss account, i.e., Development Account/Incidental Expenditure During Construction Period Account/Statement on Incidental Expenditure During Construction. The treatment of such incomes for arriving at the amount of expenditure to be capitalised/deferred, has been dealt with in para 15.2. According to para 15.2, from the total of the items of indirect expenditure (mentioned in para 15.1 e.g. preliminary project expenditure, financial expenses, depreciation on fixed assets used during the period of construction etc.), would be deducted the income, if any, earned during the period of construction, provided it can be identified with the project. Note: Currently, Guidance Note ‘Accounting Treatment of Expenditure during Construction Period’ is under revision. (c) In this case, the capital expenditure incurred by the company would not be represented by any actual assets, since the roads would remain the property of the relevant State authorities even though a part of their cost has been defrayed by the company in order to facilitate its business. Having regard to the nature of the expenditure and the purpose for which it is incurred, it is suggested in para 10 of Guidance Note on Treatment of Expenditure During Construction Period that it would be more appropriate and realistic to classify such expenditure in the balance sheet under the heading of "Capital Expenditure" rather than either, write-off the expenditure to revenue or classify the expenditure under the heading of "Miscellaneous Expenditure" or "Deferred Revenue Expenditure" subject to two conditions. In the first place, the description of the specific item on the balance sheet should be such as to indicate quite clearly that the capital expenditure is not represented by any assets owned by the company. In the second place, the capital expenditure should be written off over the approximate period of its utility or over a relatively brief period not exceeding five years, whichever is less. Question 39 A factory went into commercial production on 1st April, 1997. It uses as its raw materials product X on which excise duty of Rs. 30 per Kg. is paid and product Y on which excise duty of Rs. 20 per kg. is paid. On 31st March, 1997 it had stock of 20,000 kgs. of X and 15,000 kgs. of Y which it had purchased at an all inclusive price of Rs. 150 per kg. for X and Rs. 120 per kg. for Y. The suppliers of X and Yare to receive payment on 15th May, 1997. During April 1997, the factory manufactured 40,000 units of the end product for which the consumption of material X was 60,000 kgs. and material Y was 45,000 kgs. The excise duty on the end product is Rs. 60 per unit. 30,000 units of the end product were dispatched, 8,000 units were kept in warehouse and balance 2,000 kgs. were kept in finished goods godown. During the month the factory purchased 50,000 kgs. of X at Rs. 145 per kg. (inclusive of excise duty of Rs. 30 per kg.) on credit of 60 days and 50,000 kgs. of Y at Rs. 110 per kg. (inclusive of excise duty of Rs. 20 per kg.) on credit of 45 days. The cost of "converting" the raw materials into finished product amounts to Rs. 150 per unit of end product of which Rs. 100 is "cash cost" paid immediately and Rs. 50 represents non-cash charge for depreciation. There is no work in process. Sales are effected at Rs. 750 per unit in respect of credit transactions and at Rs. 700 per unit in respect of cash transactions. 20% of despatches were in respect of cash transactions while the balance 80% were in respect of credit transactions (one month credit). You `are required to: (a) (i) Calculate modvat credit available, modvat credit availed of and balance in modvat credit as on 30th April, 1997. (ii) Show the necessary ledger accounts in respect of modvat. (3 + 2 = 5 marks) (b) Value the inventory of: (i) raw material (ii) finished goods in warehouse (iii) finished goods in finished goods godown on "first in first out" principle. (3 marks) (c) Show the ledger accounts of customers, suppliers and bank, assuming that the necessary bank balance is available at the start of the month to meet "cash" expenses of that month. (3 marks) (d) Calculate the working capital as on 30th April, 1997. (2 marks) Accounting Theory CA Final (Existing) 53 (e) State the impact" of 'modvat' on working capital requirement of the factory as on 30th April, 1997. (2 marks) (May, 1997) Answer (a) (i) Excise duty paid on raw materials: X Y Total Kgs. @ Amount Kgs. @ Amount Amount Rs. Rs. Rs. Rs. Rs. Stock on 31st March, 1997 20,000 30 6,00,000 15,000 20 3,00,000 9,00,000 Purchases 50,000 30 15,00,000 50,000 20 10,00,000 25,00,000 21,00,000 13,00,000 34,00,000 Modvat credit available: Rs. 21,00,000 + 13,00,000 = Rs. 34,00,000 Modvat credit available of: Production = 40,000 units Excise duty on the end product = Rs. 60 per unit Modvat credit availed of = 40,000  60 = Rs. 24,00,000 Balance in Modvat credit 34,00,000 – 24,00,000 = Rs. 10,00,000 Note: Normally goods are removed from factory on payment of excise duty. However, in respect of certain goods, provision has been made to store the goods in warehouses without payment of duty (Rule 20 of Central Excise Rules, 2002). These provisions are also applicable to goods transferred to customs warehouse. It is to be noted that as per para 33 of The Guidance Note on Accounting Treatment for Excise Duty, it is necessary that a provision for liability in respect of unpaid excise duty should be made in the accounts in respect of stocks lying in the factory or warehouse since the liability for excise duty arises when the manufacture of the goods is completed. (ii) Modvat Credit Receivable Account Dr. Cr. 1997 1997 Cr. April 1 To Balance b/d April By Excise Duty A/c 24,00,000 X 6,00,000 Y 3,00,000 9,00,000 April 31 By Balance c/d 10,00,000 April To Suppliers A/c X 15,00,000 Y 10,00,000 25,00,000 ________ 34,00,000 34,00,000 Purchases Account Dr. Cr. 1997 1997 April To Suppliers A/c April 31 By Balance c/d 1,02,50,000 X: [50,000  (145 – 30)] 57,50,000 Accounting Theory CA Final (Existing) 54 Y: [50,000  (110 – 20)] 45,00,000 _________ 1,02,50,000 1,02,50,000 (b) Valuation of Inventory (i) Raw material: X Y (Kgs.) (Kgs.) Opening stock 20,000 15,000 Purchases 50,000 50,000 70,000 65,000 Consumption 60,000 45,000 Closing stock 10,000 20,000 Inventory: Rs. X : 10,000  (145 – 30) 11,50,000 Y : 20,000  (110 – 20) 18,00,000 29,50,000 (ii) Finished goods in warehouse Rs. Raw material cost of 80,000 units of output X : 12,000*  (145 – 30) 13,80,000 Y : 9,000*  (110 – 20) 8,10,000 21,90,000 Conversion cost Cash cost : 8,000  Rs. 100 8,00,000 Non-cash : 8,000  Rs. 50 4,00,000 12,00,000 Excise duty 8,000  Rs. 60 4,80,000 38,70,000 * For 40,000 units of output, input of X = 60,000 Kgs. input of Y = 45,000 Kgs. Therefore, for 8,000 units of finished goods in warehouse: Input of X = Kgs. 12,000 8,000 40,000 60,000   Accounting Theory CA Final (Existing) 55 Input of Y = Kgs. 9,000 8,000 40,000 45,000   (iii) Finished goods in finished goods godown Rs. Cost of 8,000 units of finished goods in warehouse 38,70,000 Cost of 2,000 units of finished goods in finished goods godown =   2,000 000 , 8000 , 70 , 38 9,67,500 (c) Customers Account Dr. Cr. Rs. Rs. To Sales A/c        750 30,000 10080 1,80,00,000 By Balance c/d 1,80,00,000 1,80,00,000 1,80,00,000 Suppliers Account Rs. Rs. To Balance c/d 1,75,50,000 By Balance b/d X : 20,000  Rs. 150 = 30,00,000 Y : 15,000  Rs. 120 = 18,00,000 48,00,000 By Purchases A/c 1,02,50,000 By Modvat Credit Receivable A/c X : 50,000  30 = 15,00,000 Y : 50,000  20 = 10,00,000 __________ 25,00,000 1,75,50,000 1,75,50,000 Bank Account Rs. Rs. To Balance b/d 40,00,000 By Cash Expenses (40,000  100) 40,00,000 To Sales (cash sales) A/c        700 Rs. 30,000 10020 42,00,000 ________ By Balance c/d 42,00,000 ________ 82,00,000 82,00,000 (d) Working Capital as on 30th April, 1997 Current Assets: Inventory (i) Raw materials X 11,50,000 Y 18,00,000 29,50,000 (ii) Finished goods in warehouse 38,70,000 Accounting Theory CA Final (Existing) 56 in finished goods godown 9,67,500 48,37,500 Customers 1,80,00,000 Bank balance 42,00,000 Modvat credit receivable 10,00,000 3,09,87,500 Less: Current Liabilities Sundry creditors 1,75,50,000 1,34,37,500 (e) Impact of Modvat on Working Capital Requirement Modvat has enabled (i) dispatch on sale of 30,000 units of finished product, (ii) removal of 10,000 units of finished product, without payment of a single rupee in cash. Cash outlay so saved at Rs. 60 per unit is Rs. 24,00,000. It has also ensured creation of a current asset worth Rs. 10,00,000 in Modvat Credit Receivable Account. Thus Modvat reduces the pressure on working capital. Question 40 Briefly explain as per relevant Guidance Notes: (a) HSL Ltd. is manufacturing goods for local sale and exports. As on 31st March, 2003, it has the following finished stocks in the factory warehouse: (i) Goods meant for local sale Rs. 100 lakhs (cost Rs. 75 lakhs). (ii) Goods meant for exports Rs. 50 lakhs (cost Rs. 20 lakhs). Excise duty is payable at the rate of 16%. The company’s Managing Director says that excise duty is payable only on clearance of goods and hence is not a cost. Please advise HSL using guidance note, if any issued on this, including valuation of stock. (4 marks) (May, 2003) (b) SFL Ltd. is a mutual fund. The fund values the investment on “mark to market basis”. The Accountant argues since investment are valued on the above basis there is no necessity to disclose depreciation separately in the financial statements. Do you agree? (4 marks) (May, 2003) (c) A company has given counter guarantees of Rs. 2.25 crores to various banks in respect of the guarantees given by the said banks in favour of Government authorities. Outstanding counter guarantees as at the end of financial year 2003- 2004 were Rs. 1.95 crores. How should this information be shown in the Financial Statements of the Company. (4 marks)(May, 2004) (d) A Company has its share capital divided into shares of Rs. 10 each. On 1st April, 2004 it granted 10,000 employees’ stock options at Rs. 40, when the market price was Rs. 130. The options were to be exercised between 16th December, 2004 and 15th March, 2005. The employees exercised their options for 9,500 shares only; the remaining options lapsed. The company closes its books on 31st March every year. Show Journal Entries. (6 marks) Nov. 2005) Answer (a) Guidance Note on Accounting Treatment for Excise Duty says that excise duty is a duty on manufacture or production of excisable goods in India. According to Central Excise Rules, 2002, excise duty should be collected at the time of removal of goods from factory premises or factory warehouse. The levy of excise duty is upon the manufacture or production, the collection part of it is shifted to the stage of removal. Accounting Theory CA Final (Existing) 57 Further, paragraph 23(i) of the Guidance Note makes it clear that excise duty should be considered as a manufacturing expense and like other manufacturing expenses be considered as an element of cost for inventory valuation. Therefore, in the given case of HSL Ltd., the Managing Director’s contention that “excise duty is payable only on clearance of goods and hence is not a cost is incorrect. Excise duty on the goods meant for local sales should be provided for at the rate of 16% on the selling price, that is, Rs. 100 lakhs for valuation of stock. Excise duty on goods meant for exports, should be provided for, since the liability for excise duty arises when the manufacture of the goods is completed. However, if it is assumed that all the conditions specified in Rule 19 of the Central Excise Rules, 2002 regarding export of excisable goods without payment of duty are fulfilled by HSL Ltd., excise duty may not be provided for. (b) The Guidance Note on Accounting for Investments in the Financial statements of Mutual Funds provides that the investments should be marked to market on the balance sheet date. The provision for depreciation in the value of investments should be made in the books by debiting the Revenue Account. The provision so created should be shown as a deduction from the value of investments in the balance sheet. Clause 2(i) of the Eleventh Schedule provides that “where the financial statements are prepared on a mark to market basis, there need not be a separate provision for depreciation.” However keeping in view, ‘prudence’ as a factor for preparation of financial statements and correct disclosure of the amount of depreciation on investments, the guidance note recommends that the gross value of depreciation on investments should be reflected in the Revenue Account rather than the same being netted off with the appreciation in the value of other investments. In other words, depreciation/appreciation on investments should be worked out on an individual investment basis or by category of investment basis, but not on an overall basis or by category of investment. In the given case of SFL Ltd., depreciation should be separately disclosed in the financial statements. (c) The counter guarantee given by the company is, infact, an undertaking to perform what is, in any event, the obligation of the company itself. In any case, this is a matter which is in the control of the company itself and the mere possibility of a default by the company in the future cannot be said to involve the existence of a contingent liability on the balance sheet date. Thus, as per ‘Guidance Note on Guarantees and Counter-Guarantees given by Companies’, no separate disclosure is required in respect of counter guarantees. (d) Journal Entries Particulars Dr. Cr. Rs. Rs. 2004 April 1 Employee Compensation Expense Dr. 9,00,000 To Employee Stock Options Outstanding 9,00,000 (Being grant of 10,000 stock options to employees at Rs. 40 when market price is Rs. 130) 2005 16th Dec. to 15th March Bank Dr. 3,80,000 Employee stock options outstanding Dr. 8,55,000 To Equity share capital 95,000 To Securities premium 11,40,000 (Being allotment to employees of 9,500 equity shares of Rs. 10 each at a premium of Rs. 120 per share in exercise of stock options by employees) March 16 Employee stock options outstanding Dr. 45,000 To Employee compensation expense 45,000 Accounting Theory CA Final (Existing) 58 (Being entry for lapse of stock options for 500 shares) March 31 Profit and Loss A/c Dr. 8,55,000 To Employee compensation expense 8,55,000 (Being transfer of employee compensation expense to profit and loss account) Question 41 A buyer buys a stock option of New Light Company Limited on 30 th August, 2006 with a strike price of Rs.150 per unit to be expired on September 30, 2006. The premium is Rs.10 per unit and the market lot is of 100. The margin to be paid is Rs.60 per unit. Show, how the transactions will appear in the books of the seller, when: (i) The option is settled by delivery of the Asset, and (ii) The option is settled in cash and the Index price is Rs.160 per unit. (8 Marks) (May, 2007) Answer In the Books of Seller Dr. Cr. Amount Amount Rs. Rs. At the time of inception: 30 th August Equity Stock Option Margin A/c (100 x Rs.60) Dr. 6,000 To Bank A/c 6,000 (Being the initial Margin paid on option) Bank A/c (100 x Rs.10) Dr. 1,000 To Equity Stock Option Premium A/c 1,000 (Being the premium on option collected) At the time of final settlement: Bank A/c Dr. 6,000 To Equity Stock Option Margin A/c 6,000 (Being margin on equity stock option received back on exercise/expiry of option). (i) Option is settled by delivery of asset 30 th September Bank A/c Dr. 15,000 To Equity Shares of New Light Ltd A/c 15,000 (Being shares delivered on exercise of option) Equity Stock Option Premium A/c Dr. 1,000 To Profit & Loss A/c 1,000 Accounting Theory CA Final (Existing) 59 (Being premium on option recognized as income) (ii) Option is settled in cash 30 th September Profit & Loss A/c [(160 – 150) x 100] Dr. 1,000 To Bank A/c 1,000 (Being difference in index price and strike price i.e. loss on exercise of option paid in cash) Equity Stock Option Premium A/c Dr. 1,000 To Profit & Loss A/c 1,000 (Being premium on option recognized as income) Question 42 ABC Ltd. grants 1,000 employees stock options on 1.4.2004 at Rs.40, when the market price is Rs.160. The vesting period is 2½ years and the maximum exercise period is one year. 300 unvested options lapse on 1.5.2006. 600 options are exercised on 30.6.2007. 100 vested options lapse at the end of the exercise period. Pass Journal Entries giving suitable narrations. (10 Marks)(May, 2008) Answer Journal Entries in the Books of ABC Ltd. Date Particulars Dr. (Rs.) Cr. (Rs.) 31.3.2005 Employees compensation expenses account Dr. 48,000 To Employees stock option outstanding account 48,000 (Being compensation expenses recognized in respect of the employees stock option i.e. 1,000 options granted to employees at a discount of Rs. 120 each, amortised on straight line basis over 2 2 1years) Profit and loss account Dr. 48,000 To Employees compensation expenses account 48,000 (Being expenses transferred to profit and loss account at the end of the year) 31.3.2006 Employees compensation expenses account Dr. 48,000 To Employees stock option outstanding account 48,000 (Being compensation expenses Accounting Theory CA Final (Existing) 60 recognized in respect of the employee stock option i.e. 1,000 options granted to employees at a discount of Rs. 120 each, amortised on straight line basis over 2 2 1years) Profit and loss account Dr. 48,000 To Employees compensation expenses account 48,000 (Being expenses transferred to profit and loss account at the end of the year) 31.3.2007 Employees stock option outstanding account (W.N.1) Dr. 12,000 To General Reserve account (W.N.1) 12,000 (Being excess of employees compensation expenses transferred to general reserve account) 30.6.2007 Bank A/c (600 x Rs.40) Dr. 24,000 Employee stock option outstanding account (600 x Rs.120) Dr. 72,000 To Equity share capital account (600 x Rs. 10) 6,000 To Securities premium account (600 x Rs.150) 90,000 (Being 600 employees stock option exercised at an exercise price of Rs. 40 each) 01.10.2007 Employee stock option outstanding account Dr. 12,000 To General reserve account 12,000 (Being Employees stock option outstanding A/c transferred to General Reserve A/c, on lapse of 100 options at the end of exercise of option period) Working Note: On 31.3.2007, ABC Ltd. will examine its actual forfeitures and make necessary adjustments, if any to reflect expenses for the number of options that have actually vested. 700 employees stock options have completed 2.5 years vesting period, the expense to be recognized during the year is in negative i.e. Rs. No. of options actually vested (700 x Rs.120) 84,000 Less: Expenses recognized Rs.(48,000 + 48,000) 96,000 Accounting Theory CA Final (Existing) 61 Excess expenses transferred to general reserve 12,000 Question 43 How are capital expenditures not represented by any specific or tangible assets dealt in financial statements? (5 marks) (May, 2008) Answer Sometimes circumstances force a project to incur capital expenditure which is not represented by any specific or tangible assets. For example, a project may have to pay the cost of laying pipelines in order to facilitate the supply of its products or raw materials to or from a sea port but the port trust or other similar authorities may insist that the pipelines belong to them even though the cost thereof is paid by the company. In such a case, the capital expenditure incurred by the project for the stated purpose would not be represented by any actual assets, since the pipeline would remain the property of the relevant port trust or other similar authorities even though the whole or a part of their cost may have been defrayed by the company in order to facilitate its business. In such cases the expenditure so incurred would have to be treated in the books of account as the capital expenditure. There seems to be no valid objection to disclose the expenditure under the general heading of “Capital Expenditure” subject to two conditions. In the first place the description of the specific items on the balance sheet should be such as to indicate quite clearly that the capital expenditure is not represented by any assets owned by the company. In the second place the capital expenditure should be written off over the approximate period of its utility or over a relatively brief period not exceeding five years whichever is less. In fact having regard to the nature of expenditure and purpose for which it is incurred, it would be more appropriate  and realistic to classify such expenditure in the balance sheet under the heading of “Capital Expenditure” rather than  either, write off the expenditure to revenue or classify the expenditure under  the heading of “Miscellaneous  Expenditure”.    




Comments

CAclubindia's WhatsApp Groups Link


Trending Downloads